You are on page 1of 58

IV.

JUDICIAL REVIEW

Angara vs. Electoral Commission, 63 Phil. 139


(1936)

63 Phil. 139 Political Law Judicial Review Electoral


Commission
In the elections of Sept 17, 1935, Angara, and the
respondents, Pedro Ynsua et al. were candidates voted for the
position of member of the National Assembly for the first
district of the Province of Tayabas. On Oct 7, 1935, Angara was
proclaimed as member-elect of the NA for the said district. On
November 15, 1935, he took his oath of office. On Dec 3,
1935, the NA in session assembled, passed Resolution No. 8
confirming the election of the members of the National
Assembly against whom no protest had thus far been filed. On
Dec 8, 1935, Ynsua, filed before the Electoral Commission a
Motion of Protest against the election of Angara. On Dec 9,
1935, the EC adopted a resolution, par. 6 of which fixed said
date as the last day for the filing of protests against the
election, returns and qualifications of members of the NA,
notwithstanding the previous confirmation made by the NA.
Angara filed a Motion to Dismiss arguing that by virtue of the
NA proclamation, Ynsua can no longer protest. Ynsua argued
back by claiming that EC proclamation governs and that the
EC can take cognizance of the election protest and that the EC
cannot be subject to a writ of prohibition from the SC.
ISSUES: Whether or not the SC has jurisdiction over such
matter.
Whether or not EC acted without or in excess of jurisdiction in
taking cognizance of the election protest.
HELD: The SC ruled in favor of Angara. The SC emphasized
that in cases of conflict between the several departments and
among the agencies thereof, the judiciary, with the SC as the
final arbiter, is the only constitutional mechanism devised
finally to resolve the conflict and allocate constitutional
boundaries.
That judicial supremacy is but the power of judicial review in
actual and appropriate cases and controversies, and is the

power and duty to see that no one branch or agency of the


government transcends the Constitution, which is the source
of all authority.
That the Electoral Commission is an independent
constitutional creation with specific powers and functions to
execute and perform, closer for purposes of classification to
the legislative than to any of the other two departments of the
government.
That the Electoral Commission is the sole judge of all contests
relating to the election, returns and qualifications of members
of the National Assembly.

Francisco v. House of Rep., G.R. No. 160261,


Nov.10, 2003

FACTS:
On July 22, 2002, the House of Representatives adopted a
Resolution, sponsored by Representative Felix William D.
Fuentebella, which directed the Committee on Justice "to
conduct an investigation, in aid of legislation, on the manner
of disbursements and expenditures by the Chief Justice of the
Supreme Court of the Judiciary Development Fund (JDF)." On
June 2, 2003, former President Joseph E. Estrada filed an
impeachment complaint against Chief Justice Hilario G. Davide
Jr. and seven Associate Justices of this Court for "culpable
violation of the Constitution, betrayal of the public trust and
other high crimes." The complaint was endorsed by
Representatives Rolex T. Suplico, Ronaldo B. Zamora and
Didagen Piang Dilangalen, and was referred to the House
Committee. The House Committee on Justice ruled on October
13, 2003 that the first impeachment complaint was "sufficient
in form," but voted to dismiss the same on October 22, 2003
for being insufficient in substance. To date, the Committee
Report to this effect has not yet been sent to the House in
plenary in accordance with the said Section 3(2) of Article XI of
the Constitution. Four months and three weeks since the filing
on June 2, 2003 of the first complaint or on October 23, 2003,
a day after the House Committee on Justice voted to dismiss
it, the second impeachment complaint was filed with the
Secretary General of the House by Representatives Gilberto C.
Teodoro, Jr. and Felix William B. Fuentebella against Chief
Justice Hilario G. Davide, Jr., founded on the alleged results of
the legislative inquiry initiated by above-mentioned House

Resolution. This second impeachment complaint was


accompanied by a "Resolution of Endorsement/Impeachment"
signed by at least one-third (1/3) of all the Members of the
House of Representatives.
ISSUES:
1. Whether or not the filing of the second impeachment
complaint against Chief Justice Hilario G. Davide, Jr. with the
House of Representatives falls within the one year bar
provided in the Constitution.
2. Whether the resolution thereof is a political question has
resulted in a political crisis.
HELD:
1. Having concluded that the initiation takes place by the act
of filing of the impeachment complaint and referral to the
House Committee on Justice, the initial action taken thereon,
the meaning of Section 3 (5) of Article XI becomes clear. Once
an impeachment complaint has been initiated in the foregoing
manner, another may not be filed against the same official
within a one year period following Article XI, Section 3(5) of
the Constitution. In fine, considering that the first
impeachment complaint, was filed by former President Estrada
against Chief Justice Hilario G. Davide, Jr., along with seven
associate justices of this Court, on June 2, 2003 and referred to
the House Committee on Justice on August 5, 2003, the
second impeachment complaint filed by Representatives
Gilberto C. Teodoro, Jr. and Felix William Fuentebella against
the Chief Justice on October 23, 2003 violates the
constitutional prohibition against the initiation of impeachment
proceedings against the same impeachable officer within a
one-year period.
2.From the foregoing record of the proceedings of the 1986
Constitutional Commission, it is clear that judicial power is not
only a power; it is also a duty, a duty which cannot be
abdicated by the mere specter of this creature called the
political question doctrine. Chief Justice Concepcion hastened
to clarify, however, that Section 1, Article VIII was not intended
to do away with "truly political questions." From this
clarification it is gathered that there are two species of

political questions: (1) "truly political questions" and (2) those


which "are not truly political questions." Truly political
questions are thus beyond judicial review, the reason for
respect of the doctrine of separation of powers to be
maintained. On the other hand, by virtue of Section 1, Article
VIII of the Constitution, courts can review questions which are
not truly political in nature.

PACU vs Secretary of Education, 97 Phil. 806


(1955)

FACTS:
The Philippine Association of Colleges and Universities (PACU)
assailed the constitutionality of Act No. 2706 as amended by
Act No. 3075 and Commonwealth Act No. 180. These laws
sought to regulate the ownership of private schools in the
country. It is provided by these laws that a permit should first
be secured from the Secretary of Education before a person
may be granted the right to own and operate a private school.
This also gives the Secretary of Education the discretion to
ascertain standards that must be followed by private schools.
It also provides that the Secretary of Education can and may
ban certain textbooks from being used in schools.
PACU contends that the right of a citizen to own and operate a
school is guaranteed by the Constitution, and any law
requiring previous governmental approval or permit before
such person could exercise said right, amounts to censorship
of previous restraint, a practice abhorrent to our system of law
and government. PACU also avers that such power granted to
the Secretary of Education is an undue delegation of
legislative power; that there is undue delegation because the
law did not specify the basis or the standard upon which the
Secretary must exercise said discretion; that the power to ban
books granted to the Secretary amounts to censorship.
ISSUE: Whether or not Act No, 2706 as amended is
unconstitutional.
HELD: No. In the first place, there is no justiciable controversy
presented. PACU did not show that it suffered any injury from
the exercise of the Secretary of Education of such powers
granted to him by the said law.

Second, the State has the power to regulate, in fact control,


the ownership of schools. The Constitution provides for state
control of all educational institutions even as it enumerates
certain fundamental objectives of all education to wit, the
development of moral character, personal discipline, civic
conscience and vocational efficiency, and instruction in the
duties of citizenship. The State control of private education
was intended by the organic law.
Third, the State has the power to ban illegal textbooks or
those that are offensive to Filipino morals. This is still part of
the power of control and regulation by the State over all
schools.

Mariano vs Comelec, G.R. No. 119694 March 7,


1995

FACTS:
Juanito Mariano, a resident of Makati, along with residents of
Taguig suing as taxpayers, assail Sections 2, 51 and 52 of R.A.
No. 7854 (An Act Converting the Municipality of Makati into a
Highly Urbanized City to be known as the City of Makati).
Another petition which contends the unconstitutionality of R.A.
No. 7854 was also filed by John H. Osmena as a senator,
taxpayer and concerned citizen.
ISSUES:
1. Whether Section 2 of R.A. No. 7854 delineated the land
areas of the proposed city of Makati violating sections 7
and 450 of the Local Government Code on specifying
metes and bounds with technical descriptions
2. Whether Section 51, Article X of R.A. No. 7854 collides
with Section 8, Article X and Section 7, Article VI of the
Constitution stressing that they new citys acquisition of
a new corporate existence will allow the incumbent
mayor to extend his term to more than two executive
terms as allowed by the Constitution
3. Whether the addition of another legislative district in
Makati is unconstitutional as the reapportionment
cannot be made by a special law
HELD/RULING:
1. Section 2 of R.A. No. 7854 states that:
Sec. 2. The City of Makati. The Municipality of Makati shall
be converted into a highly urbanized city to be known as the
City of Makati, hereinafter referred to as the City, which shall

comprise the present territory of the Municipality of


Makati in Metropolitan Manila Area over which it has
jurisdiction bounded on the northeast by Pasig River and
beyond by the City of Mandaluyong and the Municipality of
Pasig; on the southeast by the municipalities of Pateros and
Taguig; on the southwest by the City of Pasay and the
Municipality of Taguig; and, on the northwest, by the City of
Manila.
Emphasis has been provided in the provision under dispute.
Said delineation did not change even by an inch the land area
previously covered by Makati as a municipality. It must be
noted that the requirement of metes and bounds was meant
merely as a tool in the establishment of LGUs. It is not an end
in itself.
Furthermore, at the time of consideration or R.A. No. 7854, the
territorial dispute between the municipalities of Makati and
Taguig over Fort Bonifacio was under court litigation. Out of
becoming a sense of respect to co-equal department of
government, legislators felt that the dispute should be left to
the courts to decide.
1. Section 51 of R.A. No. 7854 provides that:
Sec. 51. Officials of the City of Makati. The represent
elective officials of the Municipality of Makati shall continue as
the officials of the City of Makati and shall exercise their
powers and functions until such time that a new election is
held and the duly elected officials shall have already qualified
and assume their offices: Provided, The new city will acquire a
new corporate existence. The appointive officials and
employees of the City shall likewise continues exercising their
functions and duties and they shall be automatically absorbed
by the city government of the City of Makati.
Section 8, Article X and section 7, Article VI of the Constitution
provide the following:
Sec. 8. The term of office of elective local officials, except
barangay officials, which shall be determined by law, shall be
three years and no such official shall serve for more than
three consecutive terms. Voluntary renunciation of the
office for any length of time shall not be considered as an
interruption in the continuity of his service for the full term for
which he was elected.
xxx xxx xxx
Sec. 7. The Members of the House of Representatives shall be
elected for a term of three years which shall begin, unless

otherwise provided by law, at noon on the thirtieth day of June


next following their election.
No Member of the House of Representatives shall serve for
more than three consecutive terms. Volunta ry renunciation of
the office for any length of time shall not be considered as an
interruption in the continuity of his service for the full term for
which he was elected.
This challenge on the controversy cannot be entertained as
the premise on the issue is on the occurrence of many
contingent events. Considering that these events may or may
not happen, petitioners merely pose a hypothetical issue
which has yet to ripen to an actual case or controversy.
Moreover, only Mariano among the petitioners is a resident of
Taguig and are not the proper parties to raise this abstract
issue.
1. Section 5(1), Article VI of the Constitution clearly
provides that the Congress may be comprised of not
more than two hundred fifty members, unless
otherwise provided by law. As thus worded, the
Constitution did not preclude Congress from increasing
its membership by passing a law, other than a general
reapportionment of the law.

Cutaran v. DENR, G.R. No. 134958, January 31,


2001

FACTS: Cutaran et. al, assails the validity of DENR Special


Order 31, Special Order 25, and Department Administrative
Order 2 for being issued without prior legislative authority.
Special Order (SO) 31 (1990): Creation of a Special Task force
on acceptance, identification, delineation and recognition of
ancestral land claims nationwide. Department Administrative
Order (DAO) 2: Implementing Rules and Guidelines of Special
Order no. 25. The same year SO 31 was issued, relatives of
petitioners filed separate applications for Certificate of
Ancestral Land Claim (CALC) for the land they occupy inside
the Camp John Hay Reservation. These petitions were denied.
Also pursuant to the SOs the heirs of A peg Cervantes filed
application for CALC for some portions of the lands in CampJohn Hay Reservation, overlapping some of the land occupied
by the petitioners. The petitioners contend that if not for the
respondents timely resistance to the Orders, the petitioners

would be totally evicted from their land. Petitioners filed in the


CA petition to enjoin respondents from implementing Orders
on the ground that they are void for lack of legal basis. Court
of Appeals ruled that SO31 has no force and effect for
preempting legislative prerogative for it was issued prior to the
effectivity of RA7586 (National Integrated Protected Systems),
but it sustained SO25 and DAO2 on the ground that they were
issued pursuant to powers delegated to DENR under RA7586.
Petitioners now contend that CA erred in upholding the validity
of SO25 and DAO2 and seek to enjoin the DENR from
processing the application of CALC of heirs of Carantes. ISSUE:
Whether or Not Special Order no. 25 and Department
Administrative Order no. 2 are valid. RULING: The Court ruled
that it is not a justiciable controversy. The petition was
prematurely filed. There is yet no justiciable controversy for
the court to resolve. The adverse legal interests involved are
the competing claims of the petitioners and heirs of Carantes
to possess a common piece of land. since the CALC application
of the heirs of Carantes has not yet been granted or issued,
and which the DENR may or may not grant, there is yet no
actual or imminent violation of petitioners asserted right to
possess the disputed land. Justiciable Controversy has been
defined as : a definite and concrete dispute touching on the
legal relations of parties having adverse legal interests which
may be resolved by a court of law through the application of a
law. Subject to certain well-defined exceptions, the courts will
not touch an issue involving the validity of a law unless there
has been a governmental act accomplished or performed that
has a direct adverse effect on the legal right of the person
contesting its validity. This Court cannot rule on the basis of
petitioners speculation that the DENR will approve the
application of the heirs of Carantes. There must be an actual
governmental act which directly causes or will imminently
cause injury to the alleged legal right of the petitioner to
possess the land before the jurisdiction of this Court may be
invoked. There is no showing that the petitioners were being
evicted from the land by the heirs of Carantes under orders
from the DENR.

Montesclaros v. Comelec, G.R. No. 152295. July 9,


2002

Facts:

Petitioners sought to prevent the postponement of the 2002


SK election to a later date since doing so may render them
unqualified to vote or be voted for in view of the age limitation
set by law for those who may participate. The SK elections was
postponed since it was deemed "operationally very difficult" to
hold both SK and Barangay elections simultaneously in May
2002. Petitioners also sought to enjoin the lowering of age for
membership in the SK.
Issue:
Whether or not there was grave abuse of discretion amounting
to lack or excess of jurisdiction imputable to respondents.
Held:
The Court held that, in the present case, there was no actual
controversy requiring the exercise of the power of judicial
review.
While seeking to prevent a postponement of the May 6, 2002
SK elections, petitioners are nevertheless amenable to a
resetting of the SK elections to any date not later than July 15,
2002. RA No. 9164 has reset the SK elections to July 15, 2002,
a date acceptable to petitioners. Under the same law,
Congress merely restored the age requirement in PD No. 684,
the original charter of the SK, which fixed the maximum age
for membership in the SK to youths less than 18 years old.
Petitioners do not have a vested right to the permanence of
the age requirement under Section 424 of the Local
Government Code of 1991.
RA 9164 which resets and prescribes the qualifications of
candidates and voters for the SK elections was held to be
applicable on the July 15 2002 election. Its constitutionality
not having been assailed in the first place.
The Court ruled that petitioners had no personal and
substantial interest in maintaining this suit, that the petition
presented no actual justiciable controversy, that petitioners
did not cite any provision of law that is alleged to be
unconstitutional, and that there was no grave abuse of
discretion on the part of public respondents.

Atlas Fertilizer v. Sec, DAR, G.R. No. 93100, June 19,


1997

Facts: Petitioner, Atlas Fertilizer engaged in the


aquaculture industry utilizing fishponds and prawn
farms. Assailed Sections 3 (b), 11, 13, 16 (d), 17 and 32
of R.A. 6657 (Comprehensive Agrarian Reform Law), as
well as the implementing guidelines and procedures
contained in Administrative Order Nos. 8 and 10 Series
of 1988 issued by public respondent Secretary of the
Department of Agrarian Reform as unconstitutional.
They contend that R.A. 6657, by including the raising of
fish and aquaculture operations including fishponds and
prawn ponds, treating them as in the same class or
classification as agriculture or farming violates the
equal protection clause of the Constitution and
therefore void.
Issue: Whether or not RA 6657 is unconstitutional.
Decision: Petition dismissed. R.A. No. 7881 approved
by Congress on 20 February 1995 expressly state that
fishponds and prawn farms are excluded from the
coverage of CARL. In view of the foregoing, the question
concerning the constitutionality of the assailed
provisions has become moot and academic with the
passage of R.A. No. 7881.
Gonzales v. Narvasa, G.R. No. 140835. August 14,
2000

Gonzales v Narvasa G.R. No. 140835, August 14, 2000


Facts:
O n De ce mb e r 9, 1999, a pe ti ti on f orp roh i b i ti on an
d m a n d a m u s w a s fi l e d a s s a i l i n g t h e constitutionality
of thecreation of the Preparatory Comission
on Constitutional Reform (PCCR) and of the positions of
presidential consultants, advisers and assistants .In his capacity as
citizen and as taxpayer, he seeks to enjoin the Commission on
Audit from passing in audit expenditures for the PCCR
and the presidential consultants, advisers and
a s s i s t a n t s . Pe t i t i o n e r a l s o
p r a y s t h a t t h e E xe c u t i v e S e c r e t a r y
b e c o m p e l l e d t h r o u g h a mandamus to furnish the
petitioner with information requesting the names of executive

officials holding multiple positions in government, copies of their


appointments and a list of the recipients of luxury vehicles seized by the
Bureau of Customs and turned over to Malacaang.
Issue:
Whether or not petitioner possesses the requisites of filing a suit as a
citizen and as taxpayer.
Ratio Decidendi:
The Court ruled that the petitioner did not have standing to
bring suit as citizen. Petitioner did not in fact show what
particularized interest they have to bring the suit. As civic leaders, they
still fall short of the requirements to maintain action.
Their interest in assailing the EO does not present to
be of a direct and personal character. Furthermore,
they do not sustain or are in immediate danger of sustaining
some direct injury as a result of its
enforcement.A s t a x p a y e r s , p e t i t i o n e r s c a n n o t a t t a c k t
h e E O. T h e r e i s n o a p p r o p r i a t i o n g r a n t e d f r o m Congr
ess but only an authorization by the president. There being
exercise by Congress of its taxing and spending power,
petitioner cannot be allowed to question the PCCRs creation.
The petitioner has failed to show that he is a real party in interest.With
regards to the petitioners request of disclosure to public
information, the Court upheld thatcitizens may invoke before
the courts the right to information. When a mandamus
proceedinginvolves the assertion of a public right, the
requirement of personal interest is satisfied by themere fact that
the petitioner is a citizen.The Supreme Court dismissed the
petition with the exception thatrespondent ExecutiveSe
cretary is ordered to furnish petitioner with the information requested.
Lacson v. Perez, G.R. No. 147780, May 10, 2001
DIGEST:
G.R. No. 147780, May 10, 2001
Power of the president to declare a state of rebellion
In quelling or suppressing the rebellion, the authorities may
only resort to warrantless arrests of persons suspected of
rebellion.

FACTS:
On May 1, 2001, President Macapagal-Arroyo, faced by an
angry and violent mob armed with explosives, firearms,
bladed weapons, clubs, stones and other deadly weapons
assaulting and attempting to break into Malacaang, issued
Proclamation No. 38 declaring that there was a state of
rebellion in the National Capital Region. She likewise issued
General Order No. 1 directing the Armed Forces of the
Philippines and the Philippine National Police to suppress the
rebellion in the National Capital Region. Warrantless arrests of
several alleged leaders and promoters of the rebellion were
thereafter effected.
Aggrieved by the warrantless arrests, and the declaration of a
state of rebellion, which allegedly gave a semblance of
legality to the arrests, the following four related petitions were
filed before the Court. Prior to resolution, the state of
rebellion was lifted in Metro Manila.
ISSUE:
Whether or not the declaration of a state of
rebellion is constitutional

RULING:
As to warrantless arrests
As to petitioners claim that the proclamation of a state
of rebellion is being used by the authorities to justify
warrantless arrests, the Secretary of Justice denies that
it has issued a particular order to arrest specific persons
in connection with the rebellion. xxx
With this declaration, petitioners apprehensions as to
warrantless arrests should be laid to rest.
In quelling or suppressing the rebellion, the authorities
may only resort to warrantless arrests of persons
suspected of rebellion, as provided under Section 5,
Rule 113 of the Rules of Court, if the circumstances so
warrant. The warrantless arrest feared by petitioners is,

thus, not based on the declaration of a state of


rebellion.

Chief has a vast intelligence network to gather


information, some of which may be classified as highly
confidential or affecting the security of the state. In the
exercise of the power to call, on-the-spot decisions may
be imperatively necessary in emergency situations to
avert great loss of human lives and mass destruction of
property. xxx

Was there violation of doctrine of separation of powers?


Petitioner Lumbao, leader of the Peoples Movement
against Poverty (PMAP), for his part, argues that the
declaration of a state of rebellion is violative of the
doctrine of separation of powers, being an
encroachment on the domain of the judiciary which has
the constitutional prerogative to determine or
interpret what took place on May 1, 2001, and that the
declaration of a state of rebellion cannot be an
exception to the general rule on the allocation of the
governmental powers.
We disagree. To be sure, section 18, Article VII of the
Constitution expressly provides that [t]he President
shall be the Commander-in-Chief of all armed forces of
the Philippines and whenever it becomes necessary, he
may call out such armed forces to prevent or suppress
lawless violence, invasion or rebellion thus, we held
in Integrated Bar of the Philippines v. Hon. Zamora,
(G.R. No. 141284, August 15, 2000):
xxx The factual necessity of calling out the armed forces
is not easily quantifiable and cannot be objectively
established since matters considered for satisfying the
same is a combination of several factors which are not
always accessible to the courts. Besides the absence of
testual standards that the court may use to judge
necessity, information necessary to arrive at such
judgment might also prove unmanageable for the
courts. Certain pertinent information necessary to arrive
at such judgment might also prove unmanageable for
the courts. Certain pertinent information might be
difficult to verify, or wholly unavailable to the courts. In
many instances, the evidence upon which the President
might decide that there is a need to call out the armed
forces may be of a nature not constituting technical
proof.
On the other hand, the President as Commander-in-

The Court, in a proper case, may look into the


sufficiency of the factual basis of the exercise of this
power. However, this is no longer feasible at this time,
Proclamation No. 38 having been lifted.

Defunis v. Odegaard, 416 U.S. 312 (1974)

Summary of DeFunis v. Odegaard, 416 U.S. 312, 94 S. Ct.


1704, 40 L. Ed. 2d 164 (1974).
Facts
Petitioner DeFunis, a white applicant to the University of
Washington law school, sued the Board of Regents of the
University of Washington in state court after he was denied
admission. DeFunis alleged that the law school discriminated
against applicants of certain races and ethnicities, including
whites, by admitting minority applicants with significantly
lower undergraduate grades and LSAT scores. DeFunis
maintained that his rejection was predicated on racial
discrimination in violation of the Equal Protection Clause of the
Fourteenth Amendment.
The District Court granted DeFunis injunctive relief and
ordered the law school to admit him. When DeFunis was in his
second year of law school, the Supreme Court of Washington
reversed, holding that the admissions policy was not
unconstitutional. The Supreme Court of the United States
granted DeFunis petition for a writ of certiorari and stayed the
judgment of the Supreme Court of Washington pending final
disposition of the case.
The case came before the Supreme Court of the United States
for a full hearing when DeFunis was in his final year of law
school. Although the law school assured that it would allow
DeFunis to graduate regardless of the Courts decision, both

parties contended that mootness did not exist to block formal


adjudication of the matter.
Issue
Can a case be adjudicated when subject matter jurisdiction is
lacking due to mootness, if adjudication of the suit would
resolve an important social issue?
Holding and Rule
No. When a federal courts determination of a legal issue is no
longer necessary to compel the result originally sought, the
case is moot and federal courts lack the power to hear it.
The constitutional basis of the mootness doctrine is found in
Article III of the Constitution which requires the existence of a
case or controversy. Thus, a real and live controversy must
exist at every stage of review.
The court held that when the original controversy has
disappeared prior to development of the suit, it is deemed
moot and a trial must not proceed for lack of subject matter
jurisdiction. That a matter deemed moot leaves an important
social issue unresolved is of no consequence.
Dissent (Douglas)
Due to the social significance of the issue involved in this case,
this matter should be adjudicated despite its apparent
mootness.
Dissent (Brennan)
Because of the social significance of the issue involved in this
case, failure to adjudicate this matter now will only result in a
future duplication of the court effort.

Acop v. Guingona, G.R. No. 134855, July 2, 2002


Facts:
On May 18, 1995, eleven (11) suspected members of
the criminal group known as the Kuratong
Baleleng gang were killed along Commonwealth Avenue
in Quezon City in an alleged shootout with the AntiBank Robbery Intelligence Task Group of the Philippine
National Police (PNP). SPO2 Eduardo delos Reyes, a
member of the Criminal Investigation Command (CIC) of
the PNP and who was one of the officers assigned to

conduct an investigation on the incident, made a public


disclosure of his findings that there was no shootout
and the eleven (11) suspected members of the gang
were summarily executed. This was attested by SPO2
Corazon dela Cruz, also a member of the CIC.
The senate conducted hearings to determine the
circumstances surrounding the subject incident and
SPO2 delos Reyes and SPO2 dela Cruz testified before
the Senate hearings. On June 2, 1995, former Senator
Raul Roco, who was then the Chairman of the Senate
Committee on Justice and Human Rights, recommended
that SPO2 delos Reyes and SPO2 dela Cruz be admitted
to the governments Witness Protection, Security and
Benefit Program. Accordingly, they were admitted into
the said Program.
Herein petitioners, in their capacity as tax payers, but
who are among the PNP officers implicated in
the alleged rubout, contend that under Sec. 3(d) for R.A.
No. 6981, law enforcement officres, like SPO2 delos
Reyes and SPO2 dela Cruz, are disqualified from being
admitted into the witness protection program even
though they may be testifying against other law
enforcement officers. Petitioners pray that the decision
of the
RTC be reversed and set aside and instead a) An
injunction be issued enjoining the Department of Justice
from continuing to provide the benefits accruing under
the Witness Protection Program to respondents SPO2
delos Reyes and SPO2 dela Cruz; b) Order the
immediate discharge of respondent SPO2 delos Reyes
and
SPO2 dela Cruz from WPP and for the latter to be
ordered to cease and desist from accepting benefits of
the
WPP; and c) Order respondent officers to return
whatever monetary benefits they have received from
the
government as a consequence of their wrongful and
illegal admission into the WPP.

Issue: Whether the petition for judicial review should


prosper.
Held:
In its comment, the Office of the Solicitor General (OSG)
claims that the petition lacks merit and that
the same has been rendered moot and academic
because the coverage of SPO2 delos Reyes and SPO2
dela
Cruz under the Program was already terminated on
December 3, 1997 and August 23, 1998, respectively,
as
evidenced by the letter of the Director of the Program
addressed to OSG, dated February 10, 1999. In their
comment, private respondents SPO2 delos Reyes and
SPO2 dela Cruz agree with OSG. Indeed, prayers a) and
b) above had been rendered moot and academic by
reason of the release of SPO2 delos Reyes and SPO2
dela
Cruz from the coverage of the Program.
However, we find it necessary to resolve the merits of
the principal issue raised for a proper
disposition of prayer c) for future guidance of both
bench and bar as to the applications of Sec. 3(d) and 4
of
R.A. No. 6981. As we have ruled in Alunan III vs. Mirasol,
276 SCRA 501 (1997), and Viola vs Alunan III
277 SCRA409 (1997), courts will decide a question
otherwise moot and academic if it is capable of
repetition, yet evading review.
.

Sanlakas v. Executive Secretary, G.R. 159085,


February 3, 2004

G.R. No. 159085, Feb. 3, 2004

The President's Commander-in-Chief Powers: (1) the calling out


power, (2) the power to suspend the privilege of the writ of
habeas corpus, and (3) the power to declare martial law
The Presidents authority to declare a state of rebellion springs
in the main from her powers as chief executive and, at the
same time, draws strength from her Commander-in-Chief
powers.
Legal significance of declaration of state of rebellion: Such a
declaration is devoid of any legal significance. For all legal
intents, the declaration is deemed not written.
Declaration of a State of Rebellion vis-a-vis Declaration of
Martial Law
FACTS:
In the wake of the Oakwood Incident, the President issued
Proc. 427 and G.O. 4, both declaring a state of rebellion and
calling out the AFP to suppress the rebellion. After hours-long
negotiations, the Oakwood occupation ended and the
president lifted the declaration of a state of rebellion.
ISSUE:

Whether or not the declaration of a state of rebellion is


constitutional
RULING:
Presidents Commander-in-Chief Powers
The above provision grants the President, as Commander-inChief, a sequence of graduated power[s]. From the most
to the least benign, these are: the calling out power, the power
to suspend the privilege of the writ of habeas corpus, and the
power to declare martial law. In the exercise of the latter two
powers, the Constitution requires the concurrence of two
conditions, namely, an actual invasion or rebellion, and that
public safety requires the exercise of such power. However, as
we observed in Integrated Bar of the Philippines v. Zamora,
[t]hese conditions are not required in the exercise of the
calling out power. The only criterion is that whenever it
becomes necessary, the President may call the armed forces
to prevent or suppress lawless violence, invasion or

rebellion.
Nevertheless, it is equally true that Section 18, Article VII does
not expressly prohibit the President from declaring a state of
rebellion. Note that the Constitution vests the President not
only with Commander-in-Chief powers but, first and foremost,
with Executive powers.
xxx the Commander-in-Chief powers are broad enough as it is
and become more so when taken together with the provision
on executive power and the presidential oath of office. Thus,
the plenitude of the powers of the presidency equips the
occupant with the means to address exigencies or threats
which undermine the very existence of government or the
integrity of the State.
xxx
Thus, the Presidents authority to declare a state of rebellion
springs in the main from her powers as chief executive and, at
the same time, draws strength from her Commander-in-Chief
powers. xxx
Declaration of State of Rebellion is Superfluity
The foregoing discussion notwithstanding, in calling out the
armed forces, a declaration of a state of rebellion is an utter
superfluity. At most, it only gives notice to the nation that such
a state exists and that the armed forces may be called to
prevent or suppress it. Perhaps the declaration may wreak
emotional effects upon the perceived enemies of the State,
even on the entire nation. But this Courts mandate is to probe
only into the legal consequences of the declaration. This Court
finds that such a declaration is devoid of any legal
significance. For all legal intents, the declaration is deemed
not written.
Should there be any confusion generated by the issuance of
Proclamation No. 427 and General Order No. 4, we clarify that,
as the dissenters in Lacson correctly pointed out, the mere
declaration of a state of rebellion cannot diminish or violate
constitutionally protected rights. Indeed, if a state of martial
law does not suspend the operation of the Constitution or

automatically suspend the privilege of the writ of habeas


corpus, then it is with more reason that a simple declaration of
a state of rebellion could not bring about these conditions. At
any rate, the presidential issuances themselves call for the
suppression of the rebellion with due regard to constitutional
rights.
But Declaration of State of Rebellion is not Declaration of
Martial Law; Exercise of Emergency Powers by President does
not necessarily follow
The argument that the declaration of a state of rebellion
amounts to a declaration of martial law and, therefore, is a
circumvention of the report requirement, is a leap of logic.
There is no indication that military tribunals have replaced civil
courts in the theater of war or that military authorities have
taken over the functions of civil government. There is no
allegation of curtailment of civil or political rights. There is no
indication that the President has exercised judicial and
legislative powers. In short, there is no illustration that the
President has attempted to exercise or has exercised martial
law powers.
Nor by any stretch of the imagination can the declaration
constitute an indirect exercise of emergency powers, which
exercise depends upon a grant of Congress pursuant to
Section 23 (2), Article VI of the Constitution:
Sec. 23. (1) .
(2) In times of war or other national emergency, the Congress
may, by law, authorize the President, for a limited period and
subject to such restrictions as it may prescribe, to exercise
powers necessary and proper to carry out a declared national
policy. Unless sooner withdrawn by resolution of the Congress,
such powers shall cease upon the next adjournment thereof.
The petitions do not cite a specific instance where the
President has attempted to or has exercised powers beyond
her powers as Chief Executive or as Commander-in-Chief. The
President, in declaring a state of rebellion and in calling out
the armed forces, was merely exercising a wedding of her
Chief Executive and Commander-in-Chief powers. These are

purely executive powers, vested on the President by Sections


1 and 18, Article VII, as opposed to the delegated legislative
powers contemplated by Section 23 (2), Article VI.

Pimentel v. Ermita, G.R. 164978, October 13, 2005

In the present case, the mootness of the petition does not bar
its resolution. The question of the constitutionality of the
Presidents appointment of department secretaries in an
acting capacity while Congress is in session will arise in every
such appointment.

G.R. No. 164978, Oct. 13, 2005

Nature of the power to appoint

Power of Appointment
Ad Interim Appointments vs. Temporary Appointments

The power to appoint is essentially executive in nature, and


the legislature may not interfere with the exercise of this
executive power except in those instances when the
Constitution expressly allows it to interfere. Limitations on the
executive power to appoint are construed strictly against the
legislature. The scope of the legislatures interference in the
executives power to appoint is limited to the power to
prescribe the qualifications to an appointive office. Congress
cannot appoint a person to an office in the guise of prescribing
qualifications to that office. Neither may Congress impose on
the President the duty to appoint any particular person to an
office.

FACTS:
This case was brought on when President Arroyo through
Executive Secretary Ermita issued appointments to
respondents as acting secretaries of their respective
departments. Several senators filed this petition in Court. After
Congress adjourned on Sept. 22, 2004, President Arroyo issued
ad interim appointments to same respondents, now as
secretaries of the departments to which they were previously
appointed in an acting capacity.
ISSUE:
Whether or not President Arroyos appointment of
respondents as acting secretaries without the consent
of the Commission on Appointments while Congress is
in session

RULING:
Mootness
The Solicitor General argues that the petition is moot
considering that President Arroyo already extended to
respondents ad interim appointments on Sept. 23, 2004,
immediately after the recess of Congress.
xxx However, as an exception to the rule on mootness, courts
will decide a question otherwise moot if it is capable of
repetition yet evading review.

However, even if the Commission on Appointments is


composed of members of Congress, the exercise of its power
is executive and not legislative. The Commission on
Appointments does not legislate when it exercises its power to
give or withhold consent to presidential appointments.
Standing of petitioners as individual members of
Congress
Considering the independence of the Commission on
Appointments from Congress, it is error for petitioners to claim
standing in the present case as members of Congress.
President Arroyos issuance of acting appointments while
Congress is in session impairs no power of Congress. Among
the petitioners, only the following are members of the
Commission on Appointments of the 13t Congress: Senator
Enrile as Minority Floor Leader, Senator Lacson as Assistant
Minority Floor Leader, and Senator Angara, Senator EjercitoEstrada, and Senator Osmena as members.
Thus, on the impairment of the prerogatives of members of

the Commission on Appointments, only Senators Enrile,


Lacson, Angara, Ejercito-Estrada, and Osmena have standing
in the present petition.
Constitutionality of President Arroyos issuance of
appointments to respondents as acting secretaries
The essence of an appointment in an acting capacity is its
temporary nature. It is a stop-gap measure intended to fill an
office for a limited time until the appointment of a permanent
occupant to the office. In case of vacancy in an office occupied
by an alter ego of the President, such as the office of a
department secretary, the President must necessarily appoint
an alter ego of her choice as acting secretary before the
permanent appointee of her choice could assume office.
Congress, through a law, cannot impose on the President the
obligation to appoint automatically the undersecretary as her
temporary alter ego. An alter ego, whether temporary or
permanent, holds a position of great trust and confidence.
Congress, in the guise of prescribing qualifications to an office,
cannot impose on the President who her alter ego should be.
The office of a department secretary may become vacant
while Congress is in session. Since a department secretary is
the alter ego of the President, the acting appointee to the
office must necessarily have the Presidents confidence. Thus,
by the very nature of the office of a department secretary, the
President must appoint in an acting capacity a person of her
choice even while Congress is in session. That person may or
may not be the permanent appointee, but practical reasons
may make it expedient that the acting appointee will also be
the permanent appointee.
Express provision of law allows President to make
acting appointment
Sec. 17, Chap. 5, Title I, Book III, EO 292 states that [t]he
President may temporarily designate an officer already in the
government service or any other competent person to perform
the functions of an office in the executive branch. Thus, the
President may even appoint in an acting capacity a person not
yet in the government service, as long as the President deems

that person competent.


But does Sec. 17 apply to appointments vested in the
President by the Constitution? Petitioners assert that it only
applies to appointments vested in the President by law.
Petitioners forget that Congress is not the only source of law.
Law refers to the Constitution, statutes or acts of Congress,
municipal ordinances, implementing rules issued pursuant to
law, and judicial decisions.
Issuance of appointments in an acting capacity is susceptible
to abuse: Petitioners fail to consider that acting appointments
cannot exceed one year. The law has incorporated this
safeguard to prevent abuses, like the use of acting
appointments as a way to circumvent confirmation by the
Commission on Appointments.
Ad interim appointments vs. appointments in an acting
capacity
Ad-interim appointments must be distinguished from
appointments in an acting capacity. Both of them are effective
upon acceptance. But ad-interim appointments are extended
only during a recess of Congress, whereas acting
appointments may be extended any time there is a vacancy.
Moreover ad-interim appointments are submitted to the
Commission on Appointments for confirmation or rejection;
acting appointments are not submitted to the Commission on
Appointments. Acting appointments are a way of temporarily
filling important offices but, if abused, they can also be a way
of circumventing the need for confirmation by the Commission
on Appointments (Bernas, 1987 Constitution: A commentary
(1996))
However, we find no abuse in the present case. The absence
of abuse is readily apparent from President Arroyos issuance
of ad interim appointments to respondents immediately upon
the recess of Congress, way before the lapse of one year

Joya vs PCGG, G.R. 96541, August 24, 1993

G.R. No. 96541, Aug. 24, 1993


Requisites for exercise of judicial review: (1) that the question
must be raised by the proper party; (2) that there must be an
actual case or controversy; (3) that the question must be
raised at the earliest possible opportunity; and, (4) that the
decision on the constitutional or legal question must be
necessary to the determination of the case itself.
LEGAL STANDING: a personal and substantial interest in the
case such that the party has sustained or will sustain direct
injury as a result of the governmental act that is being
challenged.
EXCEPTIONS TO LEGAL STANDING: Mandamus and Taxpayer's
Suits
REQUISITES FOR MANDAMUS: a writ of mandamus may be
issued to a citizen only when the public right to be enforced
and the concomitant duty of the state are unequivocably set
forth in the Constitution.
WHEN TAXPAYER SUIT MAY PROSPER: A taxpayer's suit can
prosper only if the governmental acts being questioned
involve disbursement of public funds upon the theory that the
expenditure of public funds by an officer of the state for the
purpose of administering an unconstitutional act constitutes a
misapplication of such funds, which may be enjoined at the
request of a taxpayer.
ACTUAL CONTROVERSY: one which involves a conflict of legal
rights, an assertion of opposite legal claims susceptible of
judicial resolution; the case must not be moot or academic or
based on extra-legal or other similar considerations not
cognizable by a court of justice.

FACTS:
The Republic of the Philippines through the PCGG entered into
a Consignment Agreement with Christies of New York, selling
82 Old Masters Paintings and antique silverware seized from
Malacanang and the Metropolitan Museum of Manila alleged to
be part of the ill-gotten wealth of the late Pres. Marcos, his

relatives and cronies. Prior to the auction sale, COA questioned


the Consignment Agreement, there was already opposition to
the auction sale. Nevertheless, it proceeded as scheduled and
the proceeds of $13,302,604.86 were turned over to the
Bureau of Treasury.
ISSUE:
Whether or not PCGG has jurisdiction and authority to
enter into an agreement with Christies of New York for
the sale of the artworks
RULING:
On jurisdiction of the Court to exercise judicial review
The rule is settled that no question involving the
constitutionality or validity of a law or governmental act may
be heard and decided by the court unless there is compliance
with the legal requisites for judicial inquiry, namely: that the
question must be raised by the proper party; that there must
be an actual case or controversy; that the question must be
raised at the earliest possible opportunity; and, that the
decision on the constitutional or legal question must be
necessary to the determination of the case itself. But the most
important are the first two (2) requisites.
Standing of Petitioners
On the first requisite, we have held that one having no right or
interest to protect cannot invoke the jurisdiction of the court
as party-plaintiff in an action. This is premised on Sec. 2, Rule
3, of the Rules of Court which provides that every action must
be prosecuted and defended in the name of the real party-ininterest, and that all persons having interest in the subject of
the action and in obtaining the relief demanded shall be joined
as plaintiffs. The Court will exercise its power of judicial review
only if the case is brought before it by a party who has the
legal standing to raise the constitutional or legal question.
"Legal standing" means a personal and substantial interest in
the case such that the party has sustained or will sustain
direct injury as a result of the governmental act that is being
challenged. The term "interest" is material interest, an interest
in issue and to be affected by the decree, as distinguished

from mere interest in the question involved, or a mere


incidental interest. Moreover, the interest of the party plaintiff
must be personal and not one based on a desire to vindicate
the constitutional right of some third and related party.
EXCEPTIONS TO LEGAL STANDING: Mandamus and Taxpayers
Suit:
There are certain instances however when this Court has
allowed exceptions to the rule on legal standing, as when a
citizen brings a case for mandamus to procure the
enforcement of a public duty for the fulfillment of a public
right recognized by the Constitution, and when a taxpayer
questions the validity of a governmental act authorizing the
disbursement of public funds.
Petitioners claim that as Filipino citizens, taxpayers and artists
deeply concerned with the preservation and protection of the
country's artistic wealth, they have the legal personality to
restrain respondents Executive Secretary and PCGG from
acting contrary to their public duty to conserve the artistic
creations as mandated by the 1987 Constitution, particularly
Art. XIV, Secs. 14 to 18, on Arts and Culture, and R.A. 4846
known as "The Cultural Properties Preservation and Protection
Act," governing the preservation and disposition of national
and important cultural properties. Petitioners also anchor their
case on the premise that the paintings and silverware are
public properties collectively owned by them and by the
people in general to view and enjoy as great works of art. They
allege that with the unauthorized act of PCGG in selling the art
pieces, petitioners have been deprived of their right to public
property without due process of law in violation of the
Constitution.
Petitioners' arguments are devoid of merit. They lack basis in
fact and in law. They themselves allege that the paintings
were donated by private persons from different parts of the
world to the Metropolitan Museum of Manila Foundation, which
is a non-profit and non-stock corporations established to
promote non-Philippine arts. The foundation's chairman was
former First Lady Imelda R. Marcos, while its president was
Bienvenido R. Tantoco. On this basis, the ownership of these
paintings legally belongs to the foundation or corporation or

the members thereof, although the public has been given the
opportunity to view and appreciate these paintings when they
were placed on exhibit.
Similarly, as alleged in the petition, the pieces of antique
silverware were given to the Marcos couple as gifts from
friends and dignitaries from foreign countries on their silver
wedding and anniversary, an occasion personal to them. When
the Marcos administration was toppled by the revolutionary
government, these paintings and silverware were taken from
Malacaang and the Metropolitan Museum of Manila and
transferred to the Central Bank Museum. The confiscation of
these properties by the Aquino administration however should
not be understood to mean that the ownership of these
paintings has automatically passed on the government without
complying with constitutional and statutory requirements of
due process and just compensation. If these properties were
already acquired by the government, any constitutional or
statutory defect in their acquisition and their subsequent
disposition must be raised only by the proper parties the
true owners thereof whose authority to recover emanates
from their proprietary rights which are protected by statutes
and the Constitution. Having failed to show that they are the
legal owners of the artworks or that the valued pieces have
become publicly owned, petitioners do not possess any clear
legal right whatsoever to question their alleged unauthorized
disposition.
Requisites for a Mandamus Suit
Further, although this action is also one of mandamus filed by
concerned citizens, it does not fulfill the criteria for a
mandamus suit. In Legaspi v. Civil Service Commission, this
Court laid down the rule that a writ of mandamus may be
issued to a citizen only when the public right to be enforced
and the concomitant duty of the state are unequivocably set
forth in the Constitution. In the case at bar, petitioners are not
after the fulfillment of a positive duty required of respondent
officials under the 1987 Constitution. What they seek is the
enjoining of an official act because it is constitutionally
infirmed. Moreover, petitioners' claim for the continued
enjoyment and appreciation by the public of the artworks is at
most a privilege and is unenforceable as a constitutional right

in this action for mandamus.


When a Taxpayer's Suit may prosper
Neither can this petition be allowed as a taxpayer's suit. Not
every action filed by a taxpayer can qualify to challenge the
legality of official acts done by the government. A taxpayer's
suit can prosper only if the governmental acts being
questioned involve disbursement of public funds upon the
theory that the expenditure of public funds by an officer of the
state for the purpose of administering an unconstitutional act
constitutes a misapplication of such funds, which may be
enjoined at the request of a taxpayer. Obviously, petitioners
are not challenging any expenditure involving public funds but
the disposition of what they allege to be public properties. It is
worthy to note that petitioners admit that the paintings and
antique silverware were acquired from private sources and not
with public money.
Actual Controversy
For a court to exercise its power of adjudication, there must be
an actual case of controversy one which involves a conflict
of legal rights, an assertion of opposite legal claims
susceptible of judicial resolution; the case must not be moot or
academic or based on extra-legal or other similar
considerations not cognizable by a court of justice. A case
becomes moot and academic when its purpose has become
stale, such as the case before us. Since the purpose of this
petition for prohibition is to enjoin respondent public officials
from holding the auction sale of the artworks on a particular
date 11 January 1991 which is long past, the issues
raised in the petition have become moot and academic.
At this point, however, we need to emphasize that this Court
has the discretion to take cognizance of a suit which does not
satisfy the requirements of an actual case or legal standing
when paramount public interest is involved. We find however
that there is no such justification in the petition at bar to
warrant the relaxation of the rule.

Agan v. PIATCO, G.R. No. 155001, May 5, 2003

FACTS:
On October 5, 1994, AEDC submitted an unsolicited proposal
to the Government through the DOTC/MIAA for the
development of NAIA International Passenger Terminal III (NAIA
IPT III).
DOTC constituted the Prequalification Bids and Awards
Committee (PBAC) for the implementation of the project and
submitted with its endorsement proposal to the NEDA, which
approved the project.
On June 7, 14, and 21, 1996, DOTC/MIAA caused the
publication in two daily newspapers of an invitation for
competitive or comparative proposals on AEDCs unsolicited
proposal, in accordance with Sec. 4-A of RA 6957, as
amended.
On September 20, 1996, the consortium composed of Peoples
Air Cargo and Warehousing Co., Inc. (Paircargo), Phil. Air and
Grounds Services, Inc. (PAGS) and Security Bank Corp.
(Security Bank) (collectively, Paircargo Consortium) submitted
their competitive proposal to the PBAC. PBAC awarded the
project to Paircargo Consortium. Because of that, it was
incorporated into Philippine International Airport Terminals Co.,
Inc.
AEDC subsequently protested the alleged undue preference
given to PIATCO and reiterated its objections as regards the
prequalification of PIATCO.
On July 12, 1997, the Government and PIATCO signed the
Concession Agreement for the Build-Operate-and-Transfer
Arrangement of the NAIA Passenger Terminal III (1997
Concession Agreement). The Government granted PIATCO the
franchise to operate and maintain the said terminal during the
concession period and to collect the fees, rentals and other
charges in accordance with the rates or schedules stipulated
in the 1997 Concession Agreement. The Agreement provided
that the concession period shall be for twenty-five (25) years
commencing from the in-service date, and may be renewed at

the option of the Government for a period not exceeding


twenty-five (25) years. At the end of the concession period,
PIATCO shall transfer the development facility to MIAA.

over or direct the operation of any privately owned public


utility or business affected with public interest.

Meanwhile, the MIAA which is charged with the maintenance


and operation of the NAIA Terminals I and II, had existing
concession contracts with various service providers to offer
international airline airport services, such as in-flight catering,
passenger handling, ramp and ground support, aircraft
maintenance and provisions, cargo handling and warehousing,
and other services, to several international airlines at the
NAIA.

The above provision pertains to the right of the State in times


of national emergency, and in the exercise of its police power,
to temporarily take over the operation of any business
affected with public interest. The duration of the emergency
itself is the determining factor as to how long the temporary
takeover by the government would last. The temporary
takeover by the government extends only to the operation of
the business and not to the ownership thereof. As such
the government is not required to compensate the
private entity-owner of the said business as there is no
transfer of ownership, whether permanent or temporary.
The private entity-owner affected by the temporary takeover
cannot, likewise, claim just compensation for the use of the
said business and its properties as the temporary takeover by
the government is in exercise of its police power and not of its
power of eminent domain.

On September 17, 2002, the workers of the international


airline service providers, claiming that they would lose their
job upon the implementation of the questioned agreements,
filed a petition for prohibition. Several employees of MIAA
likewise filed a petition assailing the legality of the various
agreements.
During the pendency of the cases, PGMA, on her speech,
stated that she will not honor (PIATCO) contracts which the
Executive Branchs legal offices have concluded (as) null and
void.
ISSUE:
Whether or not the State can temporarily take over a business
affected with public interest.
RULING:
Yes. PIATCO cannot, by mere contractual stipulation,
contravene the Constitutional provision on temporary
government takeover and obligate the government to
pay reasonable cost for the use of the Terminal and/or
Terminal Complex.
Article XII, Section 17 of the 1987 Constitution provides:
Section 17. In times of national emergency, when the public
interest so requires, the State may, during the emergency and
under reasonable terms prescribed by it, temporarily take

Article XII, section 17 of the 1987 Constitution envisions a


situation wherein the exigencies of the times necessitate the
government to temporarily take over or direct the operation
of any privately owned public utility or business affected with
public interest. It is the welfare and interest of the public
which is the paramount consideration in determining whether
or not to temporarily take over a particular business. Clearly,
the State in effecting the temporary takeover is exercising its
police power. Police power is the most essential, insistent,
and illimitable of powers. Its exercise therefore must not be
unreasonably hampered nor its exercise be a source of
obligation by the government in the absence of damage due
to arbitrariness of its exercise. Thus, requiring the government
to pay reasonable compensation for the reasonable use of the
property pursuant to the operation of the business
contravenes the Constitution.

CHR Employees Assoc. v. CHR, G.R. 155336, Nov.


25, 2004

G.R. No. 155336


November 25, 2004
FACTS: Congress passed RA 8522, otherwise known as the
General Appropriations Act of 1998. It provided for Special
Provisions Applicable to All Constitutional Offices Enjoying
Fiscal Autonomy. On the strength of these special provisions,
the CHR promulgated Resolution No. A98-047 adopting an
upgrading and reclassification scheme among selected
positions in the Commission.
By virtue of Resolution No. A98-062, the CHR collapsed the
vacant positions in the body to provide additional source of
funding for said staffing modification.
The CHR forwarded said staffing modification and upgrading
scheme to the DBM with a request for its approval, but the
then DBM secretary denied the request.
In light of the DBMs disapproval of the proposed personnel
modification scheme, the CSC-National Capital Region Office,
through a memorandum, recommended to the CSC-Central
Office that the subject appointments be rejected owing to the
DBMs disapproval of the plantilla reclassification.
Meanwhile, the officers of petitioner CHR-employees
association (CHREA) in representation of the rank and file
employees of the CHR, requested the CSC-Central Office to
affirm the recommendation of the CSC-Regional Office.
The CSC-Central Office denied CHREAs request in a Resolution
and reversed the recommendation of the CSC-Regional Office
that the upgrading scheme be censured. CHREA filed a motion
for reconsideration, but the CSC-Central Office denied the
same.
CHREA elevated the matter to the CA, which affirmed the
pronouncement of the CSC-Central Office and upheld the
validity of the upgrading, retitling, and reclassification scheme
in the CHR on the justification that such action is within the
ambit of CHRs fiscal autonomy.
ISSUE: Can the CHR validly implement an upgrading,
reclassification, creation, and collapsing of plantilla positions in
the Commission without the prior approval of the Department
of Budget and Management?
HELD: the petition is GRANTED, the Decision of the CA and its
are hereby REVERSED and SET ASIDE. The ruling CSC-National
Capital Region is REINSTATED. The 3 CHR Resolutions, without
the approval of the DBM are disallowed.

1. RA 6758, An Act Prescribing a Revised Compensation and


Position Classification System in the Government and For
Other Purposes, or the Salary Standardization Law, provides
that it is the DBM that shall establish and administer a
unified Compensation and Position Classification System.
The disputation of the CA that the CHR is exempt from the
long arm of the Salary Standardization Law is flawed
considering that the coverage thereof encompasses the entire
gamut of government offices, sans qualification.
This power to administer is not purely ministerial in
character as erroneously held by the CA. The word to
administer means to control or regulate in behalf of others; to
direct or superintend the execution, application or conduct of;
and to manage or conduct public affairs, as to administer the
government of the state.
2. The regulatory power of the DBM on matters of
compensation is encrypted not only in law, but in
jurisprudence as well. In the recent case of PRA v. Buag, this
Court ruled that compensation, allowances, and other benefits
received by PRA officials and employees without the requisite
approval or authority of the DBM are unauthorized and
irregular
In Victorina Cruz v. CA , we held that the DBM has the sole
power and discretion to administer the compensation and
position classification system of the national government.
In Intia, Jr. v. COA the Court held that although the charter of
the PPC grants it the power to fix the compensation and
benefits of its employees and exempts PPC from the coverage
of the rules and regulations of the Compensation and Position
Classification Office, by virtue of Section 6 of P.D. No. 1597,
the compensation system established by the PPC is,
nonetheless, subject to the review of the DBM.
(It should be emphasized that the review by the DBM of any
PPC resolution affecting the compensation structure of its
personnel should not be interpreted to mean that the DBM can
dictate upon the PPC Board of Directors and deprive the latter
of its discretion on the matter. Rather, the DBMs function is
merely to ensure that the action taken by the Board of
Directors complies with the requirements of the law,
specifically, that PPCs compensation system conforms as
closely as possible with that provided for under R.A. No. 6758.
)

3. As measured by the foregoing legal and jurisprudential


yardsticks, the imprimatur of the DBM must first be sought
prior to implementation of any reclassification or upgrading of
positions in government. This is consonant to the mandate of
the DBM under the RAC of 1987, Section 3, Chapter 1, Title
XVII, to wit:
SEC. 3. Powers and Functions. The Department of Budget
and Management shall assist the President in the preparation
of a national resources and expenditures budget, preparation,
execution and control of the National Budget, preparation and
maintenance of accounting systems essential to the budgetary
process, achievement of more economy and efficiency in the
management of government operations, administration of
compensation and position classification systems, assessment
of organizational effectiveness and review and evaluation of
legislative proposals having budgetary or organizational
implications.
Irrefragably, it is within the turf of the DBM Secretary to
disallow the upgrading, reclassification, and creation of
additional plantilla positions in the CHR based on its finding
that such scheme lacks legal justification.
Notably, the CHR itself recognizes the authority of the DBM to
deny or approve the proposed reclassification of positions as
evidenced by its three letters to the DBM requesting approval
thereof. As such, it is now estopped from now claiming that the
nod of approval it has previously sought from the DBM is a
superfluity
4. The CA incorrectly relied on the pronouncement of the CSCCentral Office that the CHR is a constitutional commission, and
as such enjoys fiscal autonomy.
Palpably, the CAs Decision was based on the mistaken
premise that the CHR belongs to the species of constitutional
commissions. But the Constitution states in no uncertain terms
that only the CSC, the COMELEC, and the COA shall be tagged
as Constitutional Commissions with the appurtenant right to
fiscal autonomy.
Along the same vein, the Administrative Code, on Distribution
of Powers of Government, the constitutional commissions shall
include only the CSC, the COMELEC, and the COA, which are
granted independence and fiscal autonomy. In contrast,
Chapter 5, Section 29 thereof, is silent on the grant of similar
powers to the other bodies including the CHR. Thus:

SEC. 24. Constitutional Commissions. The Constitutional


Commissions, which shall be independent, are the Civil Service
Commission, the Commission on Elections, and the
Commission on Audit.
SEC. 26. Fiscal Autonomy. The Constitutional Commissions
shall enjoy fiscal autonomy. The approved annual
appropriations shall be automatically and regularly released.
SEC. 29. Other Bodies. There shall be in accordance with the
Constitution, an Office of the Ombudsman, a Commission on
Human Rights, and independent central monetary authority,
and a national police commission. Likewise, as provided in the
Constitution, Congress may establish an independent
economic and planning agency.
From the 1987 Constitution and the Administrative Code, it is
abundantly clear that the CHR is not among the class of
Constitutional Commissions. As expressed in the oft-repeated
maxim expressio unius est exclusio alterius, the express
mention of one person, thing, act or consequence excludes all
others. Stated otherwise, expressium facit cessare tacitum
what is expressed puts an end to what is implied.
Nor is there any legal basis to support the contention that the
CHR enjoys fiscal autonomy. In essence, fiscal autonomy
entails freedom from outside control and limitations, other
than those provided by law. It is the freedom to allocate and
utilize funds granted by law, in accordance with law, and
pursuant to the wisdom and dispatch its needs may require
from time to time.22 In Blaquera v. Alcala and Bengzon v.
Drilon,23 it is understood that it is only the Judiciary, the CSC,
the COA, the COMELEC, and the Office of the Ombudsman,
which enjoy fiscal autonomy.
Neither does the fact that the CHR was admitted as a member
by the Constitutional Fiscal Autonomy Group (CFAG) ipso facto
clothed it with fiscal autonomy. Fiscal autonomy is a
constitutional grant, not a tag obtainable by membership.
We note with interest that the special provision under Rep. Act
No. 8522, while cited under the heading of the CHR, did not
specifically mention CHR as among those offices to which the
special provision to formulate and implement organizational
structures apply, but merely states its coverage to include
Constitutional Commissions and Offices enjoying fiscal
autonomy

All told, the CHR, although admittedly a constitutional creation


is, nonetheless, not included in the genus of offices accorded
fiscal autonomy by constitutional or legislative fiat.
Even assuming en arguendo that the CHR enjoys fiscal
autonomy, we share the stance of the DBM that the grant of
fiscal autonomy notwithstanding, all government offices must,
all the same, kowtow to the Salary Standardization Law. We
are of the same mind with the DBM on its standpoint, thusBeing a member of the fiscal autonomy group does not vest
the agency with the authority to reclassify, upgrade, and
create positions without approval of the DBM. While the
members of the Group are authorized to formulate and
implement the organizational structures of their respective
offices and determine the compensation of their personnel,
such authority is not absolute and must be exercised within
the parameters of the Unified Position Classification and
Compensation System established under RA 6758 more
popularly known as the Compensation Standardization Law.
5. The most lucid argument against the stand of respondent,
however, is the provision of Rep. Act No. 8522 that the
implementation hereof shall be in accordance with salary
rates, allowances and other benefits authorized under
compensation standardization laws.26
NOTES:
1. Respondent CHR sharply retorts that petitioner has no locus
standi considering that there exists no official written record in
the Commission recognizing petitioner as a bona fide
organization of its employees nor is there anything in the
records to show that its president has the authority to sue the
CHR.
On petitioners personality to bring this suit, we held in a
multitude of cases that a proper party is one who has
sustained or is in immediate danger of sustaining an injury as
a result of the act complained of. Here, petitioner, which
consists of rank and file employees of respondent CHR,
protests that the upgrading and collapsing of positions
benefited only a select few in the upper level positions in the
Commission resulting to the demoralization of the rank and file
employees. This sufficiently meets the injury test. Indeed, the
CHRs upgrading scheme, if found to be valid, potentially
entails eating up the Commissions savings or that portion of
its budgetary pie otherwise allocated for Personnel Services,

from which the benefits of the employees, including those in


the rank and file, are derived.
Further, the personality of petitioner to file this case was
recognized by the CSC when it took cognizance of the CHREAs
request to affirm the recommendation of the CSC-National
Capital Region Office. CHREAs personality to bring the suit
was a non-issue in the CA when it passed upon the merits of
this case. Thus, neither should our hands be tied by this
technical concern. Indeed, it is settled jurisprudence that an
issue that was neither raised in the complaint nor in the court
below cannot be raised for the first time on appeal, as to do so
would be offensive to the basic rules of fair play, justice, and
due process.
2. In line with its role to breathe life into the policy behind the
Salary Standardization Law of providing equal pay for
substantially equal work and to base differences in pay upon
substantive differences in duties and responsibilities, and
qualification requirements of the positions, the DBM, in the
case under review, made a determination, after a thorough
evaluation, that the reclassification and upgrading scheme
proposed by the CHR lacks legal rationalization.
The DBM expounded that Section 78 of the general provisions
of the General Appropriations Act FY 1998, which the CHR
heavily relies upon to justify its reclassification scheme,
explicitly provides that no organizational unit or changes in
key positions shall be authorized unless provided by law or
directed by the President. Here, the DBM discerned that there
is no law authorizing the creation of a Finance Management
Office and a Public Affairs Office in the CHR. Anent CHRs
proposal to upgrade twelve positions of Attorney VI, SG-26 to
Director IV, SG-28, and four positions of Director III, SG-27 to
Director IV, SG-28, in the Central Office, the DBM denied the
same as this would change the context from support to
substantive without actual change in functions.
This view of the DBM, as the laws designated body to
implement and administer a unified compensation system, is
beyond cavil. The interpretation of an administrative
government agency, which is tasked to implement a statute is
accorded great respect and ordinarily controls the construction
of the courts. In Energy Regulatory Board v. CA, we echoed the
basic rule that the courts will not interfere in matters which
are addressed to the sound discretion of government agencies

entrusted with the regulation of activities coming under the


special technical knowledge and training of such agencies.

Tanada vs. Tuvera, 136 SCRA 27 (1985)

G.R. No. L-63915 (146 SCRA 446) April 24, 1985


FACTS:
Petitioners sought a writ of mandamus to compel respondent
public officials to publish, and/or cause the publication in the
Official Gazette of various presidential decrees, letters of
instructions, general orders, proclamations, executive orders,
letter of implementation and administrative orders, invoking
the right to be informed on matters of public concern as
recognized by the 1973 constitution.
ISSUE:
Whether or not the publication of presidential decrees, letters
of instructions, general orders, proclamations, executive
orders, letter of implementation and administrative orders is
necessary before its enforcement.
RULING:
Article 2 of the Civil Code provides that laws shall take effect
after fifteen days following the completion of their publication
in the Official Gazette, unless it is otherwise
provided The Court has ruled that publication in the Official
Gazette is necessary in those cases where the legislation itself
does not provide for its effectivity date-for then the date of
publication is material for determining its date of effectivity,
which is the fifteenth day following its publication-but not
when the law itself provides for the date when it goes into
effect. Article 2 does not preclude the requirement of
publication in the Official Gazette, even if the law itself
provides for the date of its effectivity.
The publication of all presidential issuances of a public
nature or of general applicability is mandated by law.
Obviously, presidential decrees that provide for fines,

forfeitures or penalties for their violation or otherwise impose


a burden or. the people, such as tax and revenue measures,
fall within this category. Other presidential issuances which
apply only to particular persons or class of persons such as
administrative and executive orders need not be published on
the assumption that they have been circularized to all
concerned.
Publication is, therefore, mandatory.

Chavez v. PEA and Amari, G.R. 133250, July 9,


2002
FACTS:
In 1973, the Comissioner on Public Highways entered into a
contract to reclaim areas of Manila Bay with the Construction
and Development Corportion of the Philippines (CDCP).
PEA (Public Estates Authority) was created by President Marcos
under P.D. 1084, tasked with developing and leasing reclaimed
lands. These lands were transferred to the care of PEA under
P.D. 1085 as part of the Manila Cavite Road and Reclamation
Project (MCRRP). CDCP and PEA entered into an agreement
that all future projects under the MCRRP would be funded and
owned by PEA.
By 1988, President Aquino issued Special Patent No. 3517
transferring lands to PEA. It was followed by the transfer of
three Titles (7309, 7311 and 7312) by the Register of Deeds of
Paranaque to PEA covering the three reclaimed islands known
as the FREEDOM ISLANDS.
Subsquently, PEA entered into a joint venture agreement (JVA)
with AMARI, a Thai-Philippine corporation to develop the
Freedom Islands. Along with another 250 hectares, PEA and
AMARI entered the JVA which would later transfer said lands to
AMARI. This caused a stir especially when Sen. Maceda
assailed the agreement, claiming that such lands were part of
public domain (famously known as the mother of all scams).
Peitioner Frank J. Chavez filed case as a taxpayer praying for
mandamus, a writ of preliminary injunction and a TRO against
the sale of reclaimed lands by PEA to AMARI and from

implementing the JVA. Following these events, under President


Estradas admin, PEA and AMARI entered into an Amended JVA
and Mr. Chaves claim that the contract is null and void.
ISSUE:
w/n: the transfer to AMARI lands reclaimed or to be reclaimed
as part of the stipulations in the (Amended) JVA between
AMARI and PEA violate Sec. 3 Art. XII of the 1987 Constitution
w/n: the court is the proper forum for raising the issue of
whether the amended joint venture agreement is grossly
disadvantageous to the government.
HELD:
On the issue of Amended JVA as violating the constitution:
1. The 157.84 hectares of reclaimed lands comprising the
Freedom Islands, now covered by certificates of title in the
name of PEA, are alienable lands of the public domain. PEA
may lease these lands to private corporations but may not sell
or transfer ownership of these lands to private corporations.
PEA may only sell these lands to Philippine citizens, subject to
the ownership limitations in the 1987 Constitution and existing
laws.
2. The 592.15 hectares of submerged areas of Manila Bay
remain inalienable natural resources of the public domain until
classified as alienable or disposable lands open to disposition
and declared no longer needed for public service. The
government can make such classification and declaration only
after PEA has reclaimed these submerged areas. Only then can
these lands qualify as agricultural lands of the public domain,
which are the only natural resources the government can
alienate. In their present state, the 592.15 hectares of
submerged areas are inalienable and outside the commerce of
man.
3. Since the Amended JVA seeks to transfer to AMARI, a private
corporation, ownership of 77.34 hectares110 of the Freedom
Islands, such transfer is void for being contrary to Section 3,
Article XII of the 1987 Constitution which prohibits private
corporations from acquiring any kind of alienable land of the
public domain.
4. Since the Amended JVA also seeks to transfer to AMARI
ownership of 290.156 hectares111 of still submerged areas of
Manila Bay, such transfer is void for being contrary to Section
2, Article XII of the 1987 Constitution which prohibits the
alienation of natural resources other than agricultural lands of
the public domain.

PEA may reclaim these submerged areas. Thereafter, the


government can classify the reclaimed lands as alienable or
disposable, and further declare them no longer needed for
public service. Still, the transfer of such reclaimed alienable
lands of the public domain to AMARI will be void in view of
Section 3, Article XII of the 1987Constitution which prohibits
private corporations from acquiring any kind of alienable land
of the public domain.

KMU Labor Center vs. Garcia, G.R. 115381, Dec.


23, 1994

FACTS :
Then Secretary of DOTC, Oscar M. Orbos, issued Memorandum
Circular No. 90-395 to then LTFRB Chairman, Remedios A.S.
Fernando allowing provincial bus operators to charge
passengers rates within a range of 15% above and 15% below
the LTFRB official rate for a period of one (1) year.
This range was later increased by LTFRB thru a Memorandum
Circular No. 92-009 providing, among others, that "The
existing authorized fare range system of plus or minus 15 per
cent for provincial buses and jeepneys shall be widened to
20% and -25% limit in 1994 with the authorized fare to be
replaced by an indicative or reference rate as the basis for the
expanded fare range."
Sometime in March, 1994, private respondent PBOAP, availing
itself of the deregulation policy of the DOTC allowing provincial
bus operators to collect plus 20% and minus 25% of the
prescribed fare without first having filed a petition for the
purpose and without the benefit of a public hearing,
announced a fare increase of twenty (20%) percent of the
existing fares.
On March 16, 1994, petitioner KMU filed a petition before the
LTFRB opposing the upward adjustment of bus fares, which the
LTFRB dismissed for lack of merit.
ISSUE:
Whether or not the authority given by respondent LTFRB to
provincial bus operators to set a fare range of plus or minus

fifteen (15%) percent, later increased to plus twenty (20%)


and minus twenty-five (-25%) percent, over and above the
existing authorized fare without having to file a petition for the
purpose, is unconstitutional, invalid and illegal.
HELD:
Yes.
xxx
Under section 16(c) of the Public Service Act, the Legislature
delegated to the defunct Public Service Commission the power
of fixing the rates of public services. Respondent LTFRB, the
existing regulatory body today, is likewise vested with the
same under Executive Order No. 202 dated June 19, 1987. x x
x However, nowhere under the aforesaid provisions of law are
the regulatory bodies, the PSC and LTFRB alike, authorized to
delegate that power to a common carrier, a transport
operator, or other public service.

John Hay PAC. v. Lim, G.R. No. 119775, Oct. 24,


2003

Facts: Republic Act 7227, entitled "An Act Accellerating the


Convetsion of Military Reservations into other Productive uses,
Creating the Bases Conversion and Development Authority for
this Purpose, Providing Funds Therefor and for other
purposes," otherwise known as the "Bases Conversion and
Development Act of 1992," was enacted on 13 March 1992.
The law set out the policy of the government to accelerate the
sound and balanced conversion into alternative productive
uses of the former military bases under the 1947 PhilippinesUnited States of America Military Bases Agreement, namely,
the Clark and Subic military reservations as well as their
extensions including the John Hay Station (Camp John Hay) in
the City of Baguio. RA 7227 created the Bases Conversion and
Development Authority' (BCDA), vesting it with powers
pertaining to the multifarious aspects of carrying out the
ultimate objective of utilizing the base areas in accordance
with the declared government policy. RA 7227 likewise created
the Subic Special Economic [and Free Port] Zone (Subic SEZ)
the metes and bounds of which were to be delineated in a
proclamation to be issued by the President of the Philippines;

and granted the Subic SEZ incentives ranging from tax and
duty-free importations, exemption of businesses therein from
local and national taxes, to other hall-narks of a liberalized
financial and business climate. RA 7227 expressly gave
authority to the President to create through executive
proclamation, subject to the concurrence of the local
government units directly affected, other Special Economic
Zones (SEZ) in the areas covered respectively by the Clark
military reservation, the Wallace Air Station in San Fernando,
La Union, and Camp John Hay. On 16 August 1993, BCDA
entered into a Memorandum of Agreement and Escrow
Agreement with Tuntex (B.V.L) Co., Ltd. (TUNTEX) and
Asiaworld Internationale Group, Inc. (ASIAWORLD), private
corporations registered under the laws of the British Virgin
Islands, preparatory to the formation of a joint venture for the
development of Poro Point in La Union and Camp John Hay as
premier tourist destinations and recreation centers. 4 months
later or on 16 December 16, 1993, BCDA, TUNTEX and
ASIAWORLD executed a Joint Venture Agreements whereby
they bound themselves to put up a joint venture company
known as the Baguio International Development and
Management Corporation which would lease areas within
Camp John Hay and Poro Point for the purpose of turning such
places into principal tourist and recreation spots, as originally
envisioned by the parties under their AZemorandmn of
Agreement. The Baguio City government meanwhile passed a
number of resolutions in response to the actions taken by
BCDA as owner and administrator of Camp John Hay. By
Resolution of 29 September 1993, the Sangguniang
Panlungsod of Baguio City officially asked BCDA to exclude all
the barangays partly or totally located within Camp John Hay
from the reach or coverage of any plan or program for its
development. By a subsequent Resolution dated 19 January
1994, the sanggunian sought from BCDA an abdication, waiver
or quitclaim of its ownership over the home lots being
occupied by residents of 9 barangays surrounding the military
reservation. Still by another resolution passed on 21 February
1994, the sanggunian adopted and submitted to BCDA a 15point concept for the development of Camp John Hay. The
sanggunian's vision expressed, among other things, a kind of
development that affords protection to the environment, the
making of a family-oriented type of tourist destination, priority
in employment opportunities for Baguio residents and free

access to the base area, guaranteed participation of the city


government in the management and operation of the camp,
exclusion of the previously named nine barangays from the
area for development, and liability for local taxes of
businesses to be established within the camp." BCDA, TUNTEX
and ASIAWORLD agreed to some, but rejected or modified the
other proposals of the sanggunian." They stressed the need to
declare Camp John Hay a SEZ as a condition precedent to its
full development in accordance with the mandate of RA 7227.
On 11 May 1994, the sanggunian passed a resolution
requesting the Mayor to order the determination of realty
taxes which may otherwise be collected from real properties of
Camp John Hay. The resolution was intended to intelligently
guide the sanggunian in determining its position on whether
Camp John Hay be declared a SEZ, the sanggunian being of
the view that such declaration would exempt the camp's
property and the economic activity therein from local or
national taxation. More than a month later, however, the
sanggunian passed Resolution 255, (Series of 1994)," seeking
and supporting, subject to its concurrence, the issuance by
then President Ramos of a presidential proclamation declaring
an area of 285.1 hectares of the camp as a SEZ in accordance
with the provisions of RA 7227. Together with this resolution
was submitted a draft of the proposed proclamation for
consideration by the President. On 5 July 1994 then President
Ramos issued Proclamation 420 (series of 1994), "creating and
designating a portion of the area covered by the former Camp
John Hay as the John Hay Special Economic Zone pursuant to
Republic Act 7227." The John Hay Peoples Alternative
Coalition, et. al. filed the petition for prohibition, mandamus
and declaratory relief with prayer for a temporary restraining
order (TRO) and/or writ of preliminary injunction on 25 April
1995 challenging, in the main, the constitutionality or validity
of Proclamation 420 as well as the legality of the Memorandum
of Agreement and Joint Venture Agreement between the BCDA,
and TUNTEX and ASIAWORLD.
Issue: Whether the petitioners have legal standing in filing
the case questioning the validity of Presidential Proclamation
420.
Held: It is settled that when questions of constitutional
significance are raised, the court can exercise its power of
judicial review only if the following requisites are present: (1)
the existence of an actual and appropriate case; (2) a personal

and substantial interest of the party raising the constitutional


question; (3) the exercise of judicial review is pleaded at the
earliest opportunity; and (4) the constitutional question is the
lis mota of the case." RA 7227 expressly requires the
concurrence of the affected local government units to the
creation of SEZs out of all the base areas in the country.'" The
grant by the law on local government units of the right of
concurrence on the bases' conversion is equivalent to vesting
a legal standing on them, for it is in effect a recognition of the
real interests that communities nearby or surrounding a
particular base area have in its utilization. Thus, the interest of
petitioners, being inhabitants of Baguio, in assailing the
legality of Proclamation 420, is personal and substantial such
that they have sustained or will sustain direct injury as a result
of the government act being challenged." Theirs is a material
interest, an interest in issue affected by the proclamation and
not merely an interest in the question involved or an incidental
interest," for what is at stake in the enforcement of
Proclamation 420 is the very economic and social existence of
the people of Baguio City. Moreover, Petitioners Edilberto T.
Claravall and Lilia G. Yaranon were duly elected councilors of
Baguio at the time, engaged in the local governance of Baguio
City and whose duties included deciding for and on behalf of
their constituents the question of whether to concur with the
declaration of a portion of the area covered by Camp John Hay
as a SEZ. Certainly then, Claravall and Yaranon, as city officials
who voted against" the sanggunian Resolution No. 255 (Series
of 1994) supporting the issuance of the now challenged
Proclamation 420, have legal standing to bring the present
petition.

IBP v. Zamora, G.R. No. 141284, August 15, 2000

FACTS:
Invoking his powers as Commander-in-Chief under Sec 18, Art.
VII of the Constitution, President Estrada, in verbal directive,
directed the AFP Chief of Staff and PNP Chief to coordinate
with each other for the proper deployment and campaign for a
temporary period only. The IBP questioned the validity of the
deployment and utilization of the Marines to assist the PNP in
law enforcement.
ISSUE:

1. WoN the President's factual determination of the necessity


of calling the armed forces is subject to judicial review.
2. WoN the calling of AFP to assist the PNP in joint visibility
patrols violate the constitutional provisions on civilian
supremacy over the military.
RULING:
1. The power of judicial review is set forth in Section 1, Article
VIII of the Constitution, to wit:
Section 1. The judicial power shall be vested in one Supreme
Court and in such lower courts as may be established by law.
Judicial power includes the duty of the courts of justice to
settle actual controversies involving rights which are legally
demandable and enforceable, and to determine whether or not
there has been grave abuse of discretion amounting to lack or
excess of jurisdiction on the part of any branch or
instrumentality of the Government.
When questions of constitutional significance are raised, the
Court can exercise its power of judicial review only if the
following requisites are complied with, namely: (1) the
existence of an actual and appropriate case; (2) a personal
and substantial interest of the party raising the constitutional
question; (3) the exercise of judicial review is pleaded at the
earliest opportunity; and (4) the constitutional question is the
lis mota of the case.
2. The deployment of the Marines does not constitute a breach
of the civilian supremacy clause. The calling of the Marines in
this case constitutes permissible use of military assets for
civilian law enforcement. The participation of the Marines in
the conduct of joint visibility patrols is appropriately
circumscribed. It is their responsibility to direct and manage
the deployment of the Marines. It is, likewise, their duty to
provide the necessary equipment to the Marines and render
logistical support to these soldiers. In view of the foregoing, it
cannot be properly argued that military authority is supreme
over civilian authority. Moreover, the deployment of the
Marines to assist the PNP does not unmake the civilian
character of the police force. Neither does it amount to an
insidious incursion of the military in the task of law
enforcement in violation of Section 5(4), Article XVI of the
Constitution.

Executive Secretary v. CA, 429 SCRA 781, May 25, 2004

Facts: Republic Act 8042 (Migrant Workers and Overseas


Filipino Act of 1995) took effect on 15 July 1995. Prior to its
effectivity, Asian Recruitment Council Philippine CHaptr Inc
(ARCO-Phil) filed petition for declaratory relief. The alleged
that Section 6, subsections (a) to (m) is unconstitutional
because licensed and authorized recruitment agencies are
placed on equal footing with illegal recruiters. It contended
that while the Labor Code distinguished between recruiters
who are holders of licenses and non-holders thereof in the
imposition of penalties, Rep. Act No. 8042 does not make any
distinction. The penalties in Section 7(a) and (b) being based
on an invalid classification are, therefore, repugnant to the
equal protection clause, besides being excessive; hence, such
penalties are violative of Section 19(1), Article III of the
Constitution. In their answer to the petition, they contend that
ARCO-Phil has no legal standing, it being a non-stock, nonprofit organization; hence, not the real party-in-interest as
petitioner in the action. It is service-oriented while the
recruitment agencies it purports to represent are profitoriented.
Issue: Whether or not ARCO-Phil has legal standing to assail
RA 8042?
Decision: The modern view is that an association has
standing to complain of injuries to its members. This view
fuses the legal identity of an association with that of its
members. An association has standing to file suit for its
workers despite its lack of direct interest if its members are
affected by the action. An organization has standing to assert
the concerns of its constituents. However, the respondent has
no locus standi to file the petition for and in behalf of unskilled
workers. We note that it even failed to implead any unskilled
workers in its petition.

Kilosbayan v. Guingona, 232 SCRA 110 (1994)

Facts: On 21 October 1993, the Office of the President


announced that it had given the respondent PGMC the gosignal to operate the country's on-line lottery system and that
the corresponding implementing contract would be submitted
not later than 8 November 1993 "for final clearance and
approval by the Chief Executive."

(a) the locus standi of the petitioners, and


1.5 The Lessor is expected to submit a comprehensive
nationwide lottery development plan ('Development Plan')
which will include the game, the marketing of the games, and
the logistics to introduce the games to all the cities and
municipalities of the country within five (5) years.
1.7 The Lessor shall be selected based on its technical
expertise, hardware and software capability, maintenance
support, and financial resources. The Development Plan shall
have a substantial bearing on the choice of the Lessor. The
Lessor shall be a domestic corporation, with at least sixty
percent (60%) of its shares owned by Filipino shareholders. . .
1.8 Upon expiration of the lease, the Facilities shall be
owned by PCSO without any additional consideration. 3
The petitioners raise the following points of law to wit:
a) Under Section 1 of the Charter of the PCSO, the PCSO
is prohibited from holding and conducting Lotteries 'in
collaboration, association or joint venture with any person,
association, company or entity';
b) Under Act No. 3846 and established jurisprudence, a
Congressional franchise is required before any person may be
allowed to establish and operate said telecommunications
system;
c) Under Section 11, Article XII of the Constitution, a less
than 60% Filipino-owned and/or controlled corporation, like the
PGMC, is disqualified from operating a public service, like the
said telecommunications system; and
d) Respondent PGMC is not authorized by its charter and
under the Foreign Investment Act (R.A. No. 7042) to install,
establish and operate the on-line Lotto and
telecommunications systems."
Considering the above citizenship requirement, the PGMC
claims that the Berjaya Group "undertook to reduce its equity
stakes in PGMC to 40%," by selling 35% out of the original
75% foreign stockholdings to local investors.
Issue: In the deliberation on this case on 26 April 1994, the
issues are regarding:

(b) the legality and validity of the Contract of Lease in the


light of Section 1 of R.A. No. 1169, as amended by B.P. Blg. 42,
which prohibits the PCSO from holding and conducting
lotteries "in collaboration, association or joint venture with any
person, association, company or entity, whether domestic or
foreign."
Held: Requirement of standing was waived by the Court. (a)
WHEREFORE, the instant petition is hereby GRANTED and
the challenged Contract of Lease executed on 17 December
1993 by respondent Philippine Charity Sweepstakes Office
(PCSO) and respondent Philippine Gaming Management
Corporation (PGMC) is hereby DECLARED contrary to law and
invalid.
The Temporary Restraining Order issued on 11 April 1994 is
hereby MADE PERMANENT.
Ratio: A party's standing before this Court is a procedural
technicality which it may, in the exercise of its discretion, set
aside in view of the importance of the issues raised. In the
landmark Emergency Powers Cases, this Court brushed aside
this technicality because "the transcendental importance to
the public of these cases demands that they be settled
promptly and definitely, brushing aside, if we must,
technicalities of procedure. (a)
The actual lessor in this case is the PCSO and the subject
matter thereof is its franchise to hold and conduct lotteries
since it is, in reality, the PGMC which operates and manages
the on-line lottery system for a period of eight years.
As may be expected, it will induce in PGMC an active
interest and participation in the success of PCSO that is not
expected of an ordinary detached lessor who gets to be paid
his rental fee whether the lessee's business prospers or not.
PGMC's share in the operation depends on its own
performance and the effectiveness of its collaboration with
PCSO. Although the contract pretends otherwise, PGMC is a co-

investor with PCSO in what is practically, if not in a strictly


legal sense, a joint venture.

U.S. v. Richardson, 418 U.S. 166 (1974)

United States v. Richardson case brief


United States v. Richardson Case Summary
418 U.S. 166, 94 S. Ct. 2940, 41 L. Ed. 2d 678, 1974
Constitutional Law
PROCEDURAL HISTORY: Petitioner, the United States, sought
certiorari review of an order from the United States Court of
Appeals for the Third Circuit, which found respondent taxpayer
demonstrated standing to challenge the constitutionality of
the Central Intelligence Agency Act of 1949.
FACTS:
-The Respondent taxpayer attempted to gather information
from petitioner Government regarding detailed expenditures of
the CIA.
-Respondent sought a declaration that the Central Intelligence
Agency Act of 1949 (CIAA), was not constitutional because the
CIAA violated the federal Constitution's requirement to report
federal spending.
-The trial court granted a dismissal motion on the ground that
the respondent lacked standing and that the issue was
actually a political question.
-On appeal, the lower appellate court reversed the trial court's
ruling.
-The appellate court held that the respondent met the twotiered test for standing which required a logical link between
the respondent's status as a taxpayer & the statute, as well as
a nexus between respondent's status and a limitation on the
taxing and spending power.
HOLDING:
-On the Government's petition for certiorari review, the U.S.
Supreme Court reversed the lower court's holding, stating that
the respondent had failed to allege a direct injury and that he
failed to challenge the taxing or spending power, thus, the
respondent had no standing.
-The Respondent failed to show he suffered an injury that was
different from the one that he suffered by the public in

general.
CONCLUSION: The Court reversed the lower appellate court's
judgment.

ITF v. Comelec, G.R. No. 159139. Jan. 13, 2004

INFORMATION TECHNOLOGY FOUNDATION OF THE PHILIPPINES


, ET AL. VS. COMMISSION ON ELECTIONS, COMELEC CHAIRMAN
BENJAMIN ABALOS, SR., ET AL.
G.R. No. 159139. January 13, 2004
Facts: On June 7, 1995, Congress passed R.A. 8046 (An act
authorizing the COMELEC to conduct a nationwide
demonstration of a computerized election system and pilottest it in the March 1996 elections in the Autonomous Region
in Muslim Mindanao (ARMM) and for other purposes). On
December 22, 1997, Congress enacted R.A. 8436 (An act
authorizing the COMELEC to use an automated election
system in the May 11, 1998 national or local elections and in
subsequent national and local electoral exercises, providing
funds therefore and for other purposes).
On October 29, 2002, COMELEC adopted its Resolution 020170 a modernization program for the 2004 elections. It
resolved to conduct biddings for the three phases of its
Automated Election System: namely, Phase I-Voter
Registration and Validation System; Phase II-Automated
Counting and Canvassing System; and Phase III-Electronic
Transmissions.
President Gloria Macapagal-Arroyo issued EO No. 172, which
allocated the sum of P 2.5 billion to fund the AES for May 10,
2004 elections. She authorized the release of an additional P
500 million, upon the request of COMELEC.
The COMELEC issued an Invitation to Apply for Eligibility and
to Bid. There are 57 bidders who participated therein. The
Bids and Awards Committee (BAC) found MPC and the Total
Information Management Corporation (TIMC) eligible. Both
were referred to Technical Working Group (TWG) and the
Department of Science and Technology (DOST).
However, the DOST said in its Report on the Evaluation of
Technical Proposals on Phase II that both MPC and TIMC had
obtained a number of failed marks in technical evaluation.
Notwithstanding these failures, the COMELEC en banc issued

Resolution No. 6074, awarding the project to MPC.


Wherefore, petitioners Information Technology Foundation of
the Philippines wrote a letter to the COMELEC chairman
Benjamin Abalos, Sr. They protested the award of the contract
to respondent MPC. However in a letter-reply, the COMELEC
rejected the protest.
Issue: Whether or not the COMELEC committed grave abuse of
discretion in awarding the contract to MPC in violation of law
and in disregard of its own bidding rules and procedure.
Held: The Court has explained that COMELEC flagrantly
violated the public policy on public biddings (1) by allowing
MPC/MPEI to participate in the bidding even though it was not
qualified to do so; and (2) by eventually awarding the contract
to MPC/MPEI. It is clear that the Commission further
desecrated the law on public bidding by permitting the
winning bidder to alter the subject of the contract, in effect
allowing a substantive amendment without public bidding.

Jumamil v. Caf, G.R. 144570, September 21,


2005

Jumamil vs. Caf, et al.


[GR 144570, 21 September 2005]
Third Division, Corona (J): 4 concur
Facts: In 1989, Vivencio V. Jumamil filed before the Regional
Trial Court (RTC) of Panabo, Davao del Norte a petition for
declaratory relief with prayer for preliminary injunction and
writ of restraining order against Mayor Jose J. Cafe and the
members of the Sangguniang Bayan of Panabo, Davao del
Norte. He questioned the constitutionality of Municipal
Resolution 7, Series of 1989 (Resolution 7). Resolution 7,
enacting Appropriation Ordinance 111, provided for an initial
appropriation of P765,000 for the construction of stalls around
a proposed terminal fronting the Panabo Public Market which
was destroyed by fire. Subsequently, the petition was
amended due to the passage of Resolution 49, series of 1989
(Resolution 49), denominated as Ordinance 10, appropriating a
further amount of P1,515,000 for the construction of additional
stalls in the same public market. Prior to the passage of these

resolutions, Mayor Cafe had already entered into contracts


with those who advanced and deposited (with the municipal
treasurer) from their personal funds the sum of P40,000 each.
Some of the parties were close friends and/or relatives of Cafe,
et al. The construction of the stalls which Jumamil sought to
stop through the preliminary injunction in the RTC was
nevertheless finished, rendering the prayer therefor moot and
academic. The leases of the stalls were then awarded by
public raffle which, however, was limited to those who had
deposited P40,000 each. Thus, the petition was amended
anew to include the 57 awardees of the stalls as private
respondents. Jumamil alleges that Resolution Nos. 7 and 49
were unconstitutional because they were passed for the
business, occupation, enjoyment and benefit of private
respondents, some of which were close friends and/or relative
of the mayor and the sanggunian, who deposited the amount
of P40,000.00 for each stall, and with whom also the mayor
had a prior contract to award the would be constructed stalls
to all private respondents; that resolutions and ordinances did
not provide for any notice of publication that the special
privilege and unwarranted benefits conferred on the private
respondents may be availed of by anybody who can deposit
the amount of P40,000; and that nor there were any prior
notice or publication pertaining to contracts entered into by
public and private respondents for the construction of stalls to
be awarded to private respondents that the same can be
availed of by anybody willing to deposit P40,000.00. The
Regional Trial Court dismissed Jumamils petition for
declaratory relief with prayer for preliminary injunction and
writ of restraining order, and ordered Jumamil to pay
attorneys fees in the amount of P1,000 to each of the 57
private respondents. On appeal, and on 24 July 2000 (CA GR
CV 35082), the Court of Appeals affirmed the decision of the
trial court. Jumamil filed the petition for review on certiorari.
Issue [1]: Whether Jumamil had the legal standing to bring the
petition for declaratory relief
Held [1]: Legal standing or locus standi is a partys personal
and substantial interest in a case such that he has sustained
or will sustain direct injury as a result of the governmental act
being challenged. It calls for more than just a generalized
grievance. The term interest means a material interest, an
interest in issue affected by the decree, as distinguished from

mere interest in the question involved, or a mere incidental


interest. Unless a persons constitutional rights are adversely
affected by the statute or ordinance, he has no legal standing.
Jumamil brought the petition in his capacity as taxpayer of the
Municipality of Panabo, Davao del Norte and not in his
personal capacity. He was questioning the official acts of the
the mayor and the members of the Sanggunian in passing the
ordinances and entering into the lease contracts with private
respondents. A taxpayer need not be a party to the contract to
challenge its validity. Parties suing as taxpayers must
specifically prove sufficient interest in preventing the illegal
expenditure of money raised by taxation. The expenditure of
public funds by an officer of the State for the purpose of
executing an unconstitutional act constitutes a misapplication
of such funds. The resolutions being assailed were
appropriations ordinances. Jumamil alleged that these
ordinances were passed for the business, occupation,
enjoyment and benefit of private respondents (that is,
allegedly for the private benefit of respondents) because even
before they were passed, Mayor Cafe and private respondents
had already entered into lease contracts for the construction
and award of the market stalls. Private respondents admitted
they deposited P40,000 each with the municipal treasurer,
which amounts were made available to the municipality during
the construction of the stalls. The deposits, however, were
needed to ensure the speedy completion of the stalls after the
public market was gutted by a series of fires. Thus, the award
of the stalls was necessarily limited only to those who
advanced their personal funds for their construction. Jumamil
did not seasonably allege his interest in preventing the illegal
expenditure of public funds or the specific injury to him as a
result of the enforcement of the questioned resolutions and
contracts. It was only in the Remark to Comment he filed in
the Supreme Court did he first assert that he (was) willing to
engage in business and (was) interested to occupy a market
stall. Such claim was obviously an afterthought.
Issue [2]: Whether the rule on locus standi should be
relaxed.
Held [2]: Objections to a taxpayer's suit for lack of
sufficient personality, standing or interest are
procedural matters. Considering the importance to the
public of a suit assailing the constitutionality of a tax
law, and in keeping with the Court's duty, specially

explicated in the 1987 Constitution, to determine


whether or not the other branches of the Government
have kept themselves within the limits of the
Constitution and the laws and that they have not
abused the discretion given to them, the Supreme Court
may brush aside technicalities of procedure and take
cognizance of the suit. There being no doctrinal
definition of transcendental importance, the following
determinants formulated by former Supreme Court
Justice Florentino P. Feliciano are instructive: (1) the
character of the funds or other assets involved in the
case; (2) the presence of a clear case of disregard of a
constitutional or statutory prohibition by the public
respondent agency or instrumentality of the
government; and (3) the lack of any other party with a
more direct and specific interest in raising the questions
being raised. But, even if the Court disregards Jumamils
lack of legal standing, this petition must still fail. The
subject resolutions/ordinances appropriated a total of
P2,280,000 for the construction of the public market
stalls. Jumamil alleged that these ordinances were
discriminatory because, even prior to their enactment, a
decision had already been made to award the market
stalls to the private respondents who deposited P40,000
each and who were either friends or relatives of the
mayor or members of the Sanggunian. Jumamil asserted
that there (was) no publication or invitation to the
public that this contract (was) available to all who
(were) interested to own a stall and (were) willing to
deposit P40,000. Respondents, however, counter that
the public respondents act of entering into this
agreement was authorized by the Sangguniang Bayan
of Panabo per Resolution 180 dated 10 October 1988
and that all the people interested were invited to
participate in investing their savings. Jumamil failed to
prove the subject ordinances and agreements to be
discriminatory. Considering that he was asking the Court
to nullify the acts of the local political department of
Panabo, Davao del Norte, he should have clearly
established that such ordinances operated unfairly
against those who were not notified and who were thus
not given the opportunity to make their deposits. His
unsubstantiated allegation that the public was not

notified did not suffice. Furthermore, there was the


time-honored presumption of regularity of official duty,
absent any showing to the contrary.

Tolentino v. Comelec, 420 SCRA 438, January 21,


2004

The Constitutional Convention of 1971 scheduled an advance


plebiscite concerning only the proposal to lower the voting age
from 21 to 18. This was even before the rest of the draft of the
Constitution (then under revision) had been approved. Arturo
Tolentino then filed a motion to prohibit such plebiscite.
ISSUE: Whether or not the petition will prosper.

order for violating the right to privacy. He contends that the


order must be invalidated on two constitutional grounds, (1)
that it is a usurpation of the power to legislate; and (2) that it
intrudes the citizens right to privacy.
Issue: Whether or not Senator Ople has standing to maintain
suit?
Decision: Petitioner, Senator Ople is a distinguished member
of the Senate. As a Senator, petitioner is possessed of the
requisite standing to bring suit raising the issue that the issue
of Administrative Order No 308 is a usurpation of legislative
power. Oples concern that the Executive branch not to
trespass on the lawmaking domain of Congress is
understandable. The blurring demarcation line between the
power of legislature to make laws and the power of executive
to execute laws will disturb their delicate balance and cannot
be allowed.

HELD: Yes. If the advance plebiscite will be allowed, there will


be an improper submission to the people. Such is not allowed.
The proposed amendments shall be approved by a majority of
the votes cast at an election at which the amendments are
submitted to the people for ratification. Election here is
singular which meant that the entire constitution must be
submitted for ratification at one plebiscite only. Furthermore,
the people were not given a proper frame of reference in
arriving at their decision because they had at the time no idea
yet of what the rest of the revised Constitution would
ultimately be and therefore would be unable to assess the
proposed amendment in the light of the entire document. This
is the Doctrine of Submission which means that all the
proposed amendments to the Constitution shall be presented
to the people for the ratification or rejection at the same time,
NOT piecemeal.

Ople v. Torres, 293 SCRA 141 (1998)

Facts: Administrative Order No 308, otherwise known as


Adoption of a National Computerized Identification Reference
System was issued by President Fidel Ramos on 12 December
1996. Senator Blas Ople filed a petition to invalidate the said

People v. Vera, 65 Phil 56, November 16, 1937

65 Phil. 56 Political Law Constitutional Law Bill of Rights


Equal Protection Probation Law
Separation of Powers Undue Delegation of Powers Power to
Pardon
Constitutionality of Laws May the State Question Its Own
Laws
In 1934, Mariano Cu Unjieng was convicted in a criminal case
filed against him by the Hongkong and Shanghai Banking
Corporation (HSBC). In 1936, he filed for probation. The matter
was referred to the Insular Probation Office which
recommended the denial of Cu Unjiengs petition for
probation. A hearing was set by Judge Jose Vera concerning
the petition for probation. The Prosecution opposed the
petition. Eventually, due to delays in the hearing, the
Prosecution filed a petition for certiorari with the Supreme
Court alleging that courts like the Court of First Instance of
Manila (which is presided over by Judge Vera) have no
jurisdiction to place accused like Cu Unjieng under probation
because under the law (Act No. 4221 or The Probation Law),

probation is only meant to be applied in provinces with


probation officers; that the City of Manila is not a province,
and that Manila, even if construed as a province, has no
designated probation officer hence, a Manila court cannot
grant probation.

1. Yes. There is no law which prohibits the State, or its duly


authorized representative, from questioning the validity of a
law. Estoppel will also not lie against the State even if it had
been using an invalid law.
2. No, Act 4221 or the [old] Probation Law is unconstitutional.

Meanwhile, HSBC also filed its own comment on the matter


alleging that Act 4221 is unconstitutional for it violates the
constitutional guarantee on equal protection of the laws. HSBC
averred that the said law makes it the prerogative of provinces
whether or nor to apply the probation law if a province
chooses to apply the probation law, then it will appoint a
probation officer, but if it will not, then no probation officer will
be appointed hence, that makes it violative of the equal
protection clause.

Violation of the Equal Protection Clause


The contention of HSBC and the Prosecution is well taken on
this note. There is violation of the equal protection
clause. Under Act 4221, provinces were given the option to
apply the law by simply providing for a probation officer. So if
a province decides not to install a probation officer, then the
accused within said province will be unduly deprived of the
provisions of the Probation Law.
Undue Delegation of Legislative Power

Further, HSBC averred that the Probation Law is an undue


delegation of power because it gave the option to the
provincial board to whether or not to apply the probation law
however, the legislature did not provide guidelines to be
followed by the provincial board.

There is undue delegation of legislative power. Act 4221


provides that it shall only apply to provinces where the
respective provincial boards have provided for a probation
officer. But nowhere in the law did it state as to what standard
(sufficient standard test) should provincial boards follow in
determining whether or not to apply the probation law in their
province. This only creates a roving commission which will act
arbitrarily according to its whims.

Further still, HSBC averred that the Probation Law is an


encroachment of the executives power to grant pardon. They
say that the legislature, by providing for a probation law, had
in effect encroached upon the executives power to grant
pardon. (Ironically, the Prosecution agreed with the issues
raised by HSBC ironic because their main stance was the
non-applicability of the probation law only in Manila while
recognizing its application in provinces).

Encroachment of Executive Power


Though Act 4221 is unconstitutional, the Supreme Court
recognized the power of Congress to provide for probation.
Probation does not encroach upon the Presidents power to
grant pardon. Probation is not pardon. Probation is within the
power of Congress to fix penalties while pardon is a power of
the president to commute penalties.

For his part, one of the issues raised by Cu Unjieng is that, the
Prosecution, representing the State as well as the People of
the Philippines, cannot question the validity of a law, like Act
4221, which the State itself created. Further, Cu Unjieng also
castigated the fiscal of Manila who himself had used the
Probation Law in the past without question but is now
questioning the validity of the said law (estoppel).

ISSUE:
1. May the State question its own laws?

Estrada v. Sandiganbayan, G.R. No. 148560, Nov.


19, 2001

G.R. No. 148560 November 19,2001

2. Is Act 4221 constitutional?


HELD:

Lessons Applicable:
Consti Overbreadth doctrine, void-for-vagueness doctrine

Crim Law 1- mala in se


Crim pro proof beyond reasonable doubt
Laws Applicable: Art. 3 RPC
FACTS:
An information is filed against former President Joseph
Ejercito Estrada a.k.a. 'Asiong Salonga' and 'Jose
Velarde,' together with Jose 'Jinggoy' Estrada, Charlie
'Atong' Ang, Edward Serapio, Yolanda T. Ricaforte, Alma
Alfaro, John Doe a.k.a. Eleuterio Tan or Eleuterio Ramos
Tan or Mr. Uy, Jane Doe a.k.a. Delia Rajas and John Does
& Jane Does of the crime of Plunder under RA 7080 (An
Act Defining and Penalizing the Crime of Plunder)

June, 1998 to January 2001: Estrada himself


and/or in connivance/conspiracy with his co-accused,
who are members of his family, relatives by affinity or
consanguinity, business associates, subordinates and/or
other persons, by taking undue advantage of his official
position, authority, relationship, connection, or
influence, did then and there willfully, unlawfully and
criminally amass, accumulate and acquire by himself,
directly or indirectly, ill-gotten wealth of
P4,097,804,173.17 thereby unjustly enriching himself or
themselves at the expense and to the damage of the
Filipino people and the Republic of the Philippines,
through any or a combination or a series of overt or
criminal acts, or similar schemes or means

Received P545,000,000.00 in the form of gift,


share, percentage, kickback or any form of pecuniary
benefit, by himself and/or in connection with co-accused
Charlie 'Atong' Ang, Jose 'Jinggoy' Estrada, Yolanda T.
Ricaforte, Edward Serapio, and John Does and Jane
Does, in consideration of toleration or protection of
illegal gambling

Diverting, receiving, misappropriating, converting


or misusing directly or indirectly, for his or their
personal gain and benefit, public funds of
P130,000,000.00, more or less, representing a portion
of P200,000,000.00) tobacco excise tax share allocated
for the province of Ilocos Sur under R.A. No. 7171, by
himself and/or in connivance with co-accused Charlie
'Atong' Ang, Alma Alfaro, John Doe a.k.a. Eleuterio

1.
2.
3.

1.
2.
3.
4.
5.

Ramos Tan or Mr. Uy, Jane Doe a.k.a. Delia Rajas, and
other John Does & Jane Does

For His Personal Gain And Benefit, The


Government Service Insurance System (GSIS) To
Purchase 351,878,000 Shares Of Stocks, More Or Less,
And The Social Security System (SSS), 329,855,000
Shares Of Stock, More Or Less, Of The Belle Corporation
worth P1,102,965,607.50 and P744,612,450.00
respectively and by collecting or receiving, directly or
indirectly, by himself and/or in connivance with John
Does and Jane Does, commissions or percentages by
reason of said purchases which became part of the
deposit in the equitable-pci bank under the account
name Jose Velarde

by unjustly enriching himself from commissions,


gifts, shares, percentages, kickbacks, or any form of
pecuniary benefits, in connivance with John Does and
Jane Does, P3,233,104,173.17 and depositing the same
under his account name Jose Velarde at the EquitablePci Bank
Estrada questions the constitutionality of the Plunder
Law since for him:
it suffers from the vice of vagueness
it dispenses with the "reasonable doubt" standard in criminal
prosecutions
it abolishes the element of mens rea in crimes already
punishable under The Revised Penal Code
April 4, 2001: Office of the Ombudsman filed before the
Sandiganbayan 8 separate Informations, docketed as:
Crim. Case No. 26558, for violation of RA 7080, as amended
by RA 7659
Crim. Cases Nos. 26559 to 26562, inclusive, for violation of
Secs. 3, par. (a), 3, par. (a), 3, par. (e) and 3, par. (e), of RA
3019 (Anti-Graft and Corrupt Practices Act), respectively
Crim. Case No. 26563, for violation of Sec. 7, par. (d), of RA
6713 (The Code of Conduct and Ethical Standards for Public
Officials and Employees)
Crim. Case No. 26564, for Perjury (Art. 183 of The Revised
Penal Code)
Crim. Case No. 26565, for Illegal Use Of An Alias (CA No. 142,
as amended by RA 6085)
April 11, 2001: Estrada filed an Omnibus Motion on the
grounds of lack of preliminary investigation,

reconsideration/reinvestigation of offenses and opportunity to


prove lack of probable cause. - Denied

April 25, 2001: Sandiganbayan issued a Resolution in Crim.


Case No. 26558 finding that a probable cause for the offense
of plunder exists to justify the issuance of warrants for the
arrest of the accused

June 14, 2001: Estrada moved to quash the Information in


Crim. Case No. 26558 on the ground that the facts alleged
therein did NOT constitute an indictable offense since the law
on which it was based was unconstitutional for vagueness and
that the Amended Information for Plunder charged more than
1 offense Denied

Estrada filed a petition for certiorari are:


1. The Plunder Law is unconstitutional for being vague
2. The Plunder Law requires less evidence for proving the
predicate crimes of plunder and therefore violates the rights of
the accused to due process
3. Whether Plunder as defined in RA 7080 is a malum
prohibitum, and if so, whether it is within the power of
Congress to so classify it
ISSUES:
1. W/N the Plunder Law is constitutional (consti1)
2. W/N the Plunder Law dispenses with the "reasonable doubt"
standard in criminal prosecutions (crim pro)
3. W/N the Plunder Law is a malum prohibitum (crim law 1)
1.

HELD: Petition is dismissed. Plunder Law is constitutional.


YES
Miserably failed in the instant case to discharge his burden
and overcome the presumption of constitutionality of the
Plunder Law
Plunder Law contains ascertainable standards and welldefined parameters which would enable the accused to
determine the nature of his violation.
Combination- at least two (2) acts falling under different
categories of enumeration
series - must be two (2) or more overt or criminal acts
falling under the same category of enumeration
pattern - at least a combination or series of overt or
criminal acts enumerated in subsections (1) to (6) of Sec. 1 (d)
Void-For-Vagueness Doctrine - a statute which either forbids
or requires the doing of an act in terms so vague that men of

common intelligence must necessarily guess at its meaning


and differ as to its application, violates the first essential of
due process of law
o The test in determining whether a criminal statute is void for
uncertainty is whether the language conveys a sufficiently
definite warning as to the proscribed conduct when measured
by common understanding and practice
o can only be invoked against that specie of legislation that is
utterly vague on its face, i.e., that which cannot be clarified
either by a saving clause or by construction
o a statute or act may be said to be vague when it lacks
comprehensible standards that men of common intelligence
must necessarily guess at its meaning and differ in its
application.
o the statute is repugnant to the Constitution in 2 respects:
a. it violates due process for failure to accord persons,
especially the parties targeted by it, fair notice of what
conduct to avoid
b. it leaves law enforcers unbridled discretion in carrying out its
provisions and becomes an arbitrary flexing of the
Government muscle
o As for the vagueness doctrine, it is said that a litigant may
challenge a statute on its face only if it is vague in all its
possible applications

Overbreadth Doctrine - a governmental purpose may NOT


be achieved by means which sweep unnecessarily broadly and
thereby invade the area of protected freedoms
o overbreadth claims, if entertained at all, have been curtailed
when invoked against ordinary criminal laws that are sought to
be applied to protected conduct

A facial challenge is allowed to be made to a vague statute


and to one which is overbroad because of possible "chilling
effect" upon protected speech.

Criminal statutes have general in terrorem effect resulting


from their very existence, and, if facial challenge is allowed for
this reason alone, the State may well be prevented from
enacting laws against socially harmful conduct. In the area of
criminal law, the law cannot take chances as in the area of
free speech.

The overbreadth and vagueness doctrines then have


special application only to free speech cases.

2. NO.

The use of the "reasonable doubt" standard is


indispensable to command the respect and confidence of the
community in the application of criminal law.
o has acquired such exalted stature in the realm of
constitutional law as it gives life to the Due Process Clause
which protects the accused against conviction except upon
proof beyond reasonable doubt of every fact necessary to
constitute the crime with which he is charged

What the prosecution needs to prove beyond reasonable


doubt is only a number of acts sufficient to form a combination
or series which would constitute a pattern and involving an
amount of at least P50,000,000.00. There is no need to prove
each and every other act alleged in the Information to have
been committed by the accused in furtherance of the overall
unlawful scheme or conspiracy to amass, accumulate or
acquire ill-gotten wealth
o Pattern is merely a by-product of the proof of the predicate
acts. This conclusion is consistent with reason and common
sense. There would be no other explanation for a
combination or series of overt or criminal acts to stash
P50,000,000.00 or more, than "a scheme or conspiracy to
amass, accumulate or acquire ill gotten wealth."
3. NO

plunder is a malum in se which requires proof of criminal


intent (mens rea)
o Any person who participated with the said public officer in the
commission of an offense contributing to the crime of plunder
shall likewise be punished for such offense.
o In the imposition of penalties, the degree of participation and
the attendance of mitigating and extenuating circumstances,
as provided by the Revised Penal Code, shall be considered by
the court.
indicates quite clearly that mens rea is an element of plunder
since the degree of responsibility of the offender is determined
by his criminal intent
o The legislative declaration in R.A. No. 7659 that plunder is a
heinous offense implies that it is a malum in se. For when the
acts punished are inherently immoral or inherently wrong,
they are mala in se and it does not matter that such acts are
punished in a special law, especially since in the case of
plunder the predicate crimes are mainly mala in se

Umali v. Guingona, 305 SCRA 533 (1999)

Facts: Osmundo Umali the petitioner was appointed


Regional Director of the Bureau of Internal Revenue by
Pres Fidel V. Ramos. He assigned him in Manila,
November 29, 1993 to March 15, 1994 and Makati,
March 16, 1994 to August 4, 1994. On August 1, 1994,
President Ramos received a confidential memorandum
against the petitioner for alleged violations of internal
revenue laws, rules and regulations during his
incumbency as Regional Director, more particularly the
following malfeasance, misfeasance and nonfeasance.
upon receipt of the said confidential memorandum,
former President authorized the issuance of an Order for
the preventive suspension of the petitioner and
immediately referred the Complaint against the latter to
the Presidential Commission on Anti-Graft and
Corruption (PCAGC), for investigation. Petitioner was
duly informed of the charges against him. And was
directed him to send in his answer, copies of his
Statement of Assets, and Liabilities for the past three
years (3), and Personal Data Sheet. Initial hearing was
set on August 25, 1994, at 2:00 p.m., at the PCAGC
Office. On August 23, the petitioner filed his required
answer. After evaluating the evidence on record, the
PCAGC issued its Resolution of September 23, 1994,
finding a prima facie evidence to support six (6) of the
twelve (12) charges against petitioner. On October 6,
1994, acting upon the recommendation of the PCAGC,
then President Ramos issued Administrative Order No.
152 dismissing petitioner from the service, with
forfeiture of retirement and all benefits under the law.
Issues:
(1) Whether or Not AO No. 152 violated petitioner's
Right to Security of Tenure.
(2) Whether or Not Petitioner was denied due process of
law

the case under consideration, the administrative action


against the petitioner was taken prior to the institution
of the criminal case. The charges included in
Administrative Order No. 152 were based on the results
of investigation conducted by the PCAGC and not on the
criminal charges before the Ombudsman. In sum, the
petition is dismissable on the ground that the Issue
posited by the petitioner do not constitute a valid legal
basis for overturning the finding and conclusion arrived
at by the Court of Appeals. However, taking into
account the antecedent facts and circumstances
aforementioned, the Court, in the exercise of its equity
powers, has decided to consider the dismissal of the
charges against petitioner before the Ombudsman, the
succinct and unmistakable manifestation by the
Commissioner of the Bureau of Internal Revenue that
his office is no longer interested in pursuing the case,
and the position taken by the Solicitor General, that
there is no more basis for Administrative Order No. 152,
as effective and substantive supervening events that
cannot be overlooked.

(3) Whether or Not the PCAGC is a validly Constituted


government agency and whether the petitioner can
raise the issue of constitutionality belatedly in its
motion for reconsideration of the trial courts decision.
(4) Whether or Not the ombudsman's resolution
dismissing the charges against the petitioner is still
basis for the petitioner's dismissal with forfeiture of
benefits as ruled in AO No. 152
Held: Petitioner maintains that as a career executive
service officer, he can only be removed for cause and
under the Administrative Code of 1987, 6 loss of
confidence is not one of the legal causes or grounds for
removal. Consequently, his dismissal from office on the
ground of loss confidence violated his right to security
of tenure, petitioner theorized. After a careful study, we
are of the irresistible conclusion that the Court of
Appeals ruled correctly on the first three Issue. To be
sure, petitioner was not denied the right to due process
before the PCAGC. Records show that the petitioner filed
his answer and other pleadings with respect to his
alleged violation of internal revenue laws and
regulations, and he attended the hearings before the
investigatory body. It is thus decisively clear that his
protestation of non-observance of due process is devoid
of any factual or legal basis. Neither can it be said that
there was a violation of what petitioner asserts as his
security of tenure. According to petitioner, as a Regional
Director of Bureau of Internal Revenue, he is CESO
eligible entitled to security of tenure. However,
petitioner's claim of CESO eligibility is anemic of
evidentiary support. It was incumbent upon him to
prove that he is a CESO eligible but unfortunately, he
failed to adduce sufficient evidence on the matter. His
failure to do so is fatal. As regards the issue of
constitutionality of the PCAGC, it was only posed by
petitioner in his motion for reconsideration before the
Regional Trial Court of Makati. It was certainly too late to
raise for the first time at such late stage of the
proceedings. As to last issue, It is worthy to note that in

Arceta v. Mangrobang, G.R. No. 152895. June 15,


2004

Facts: On 16 September 1998, Ofelia Arceta issued a check to


Oscar Castro payable to cash for the amount of Php 740k even
with full knowledge that the account has no sufficient fund for
the said amount. The check was subsequently dishonoured by
the bank. The City Prosecutor of Metro Manila charged Arceta
of violating BP Blg 22 (Bouncing Checks Law). She did not
moved for the charge to be dismissed and pleaded not guilty.
She then petitioned for certiorari, prohibition and mandamus
assailing the constitutionality of BP Blg 22 citing the Lozano
doctrine.
Issue: Whether or not the constitutionality of BP Blg 22 is the
lis mota of the case?
Decision: Petition dismissed for utter lack of merit. Every law
has in its favour the presumption of constitutionality. To justify
its nullification there must be a clear and unequivocal breach

of the constitution and not one that is speculative, doubtful or


argumentative. Petitioner failed to show that BP Blg 22 by
itself or by implementation transgressed a provision of the
Constitution.

Under PD579, which provides that the balance of


the proceeds of sugar
trading operations shall be set aside by the
Philippine Exchange Company
as profits which shall be paid to a special fund..,
neither PNB nor PHILEX
could retain any difference claimed by the
Mirasols in the price of sugar
sold. there was nothing with which PNB was
supposed to have off-set the
Mirasols indebtedness.
(2) Compensation cannot take place where one
claim, as in the instant case,
is still the subject of litigation as the same cannot
be deemed liquidated.
(see Art 1279[4])

Mirasol V. CA
o , G.R. No. 128448, February 1, 2001
BRIEF: Under a crop loan financing scheme for
crop years 1973-1974 and 19741975, the Mirasols signed a Chattel Mortgage
which empowered PNB to sell
their sugar and to apply the proceeds as payment
of their obligations to PNB.
Thereafter, petitioners continued to avail of other
loans from and make unfunded
withdrawals from their accounts with PNB. PNB
foreclosed their mortgaged
properties for their failure to pay. Petitioners
claim that the foreclosure is invalid
since their debt has been fully paid by virtue of
legal compensation, that is, it
should be offset by the amount PNB owes them
from the sale of sugar. PNB
contends that under PD 579 issued by Marcos in
1974, all earnings from the
export sales of sugar pertained to the National
Government and were subject to
the Presidents disposition for public purposes.
The Court ruled that legal
compensation cannot take place since the
requisites for which have not been
met; the foreclosure is valid.
RULING: No, the CA did not err in upholding the
validity of the foreclosure on
petitioners property. Set-off or compensation
cannot take place because:
(1) neither of the parties are mutually creditors
and debtors of each other.
(see Art 1279[1])

Mitra vs Comelec, 104 SCRA 58 (1981)


FACTS:
We resolve the Motion for Reconsideration Filed
by public respondent Commission on Elections
(COMELEC) and the Motion for Reconsideration
with Motion for Oral Arguments filed by private
respondents Antonio V. Gonzales and Orlando R.
Balbon, Jr. (private respondents), dated July 19,
2010 and July 20, 2010, respectively, addressing
our Decision of July 2, 2010 (July 2, 2010 Decision
or Decision).We annulled in this Decision the
February 10, 2010 and May 4, 2010 Resolutions of
the COMELEC, and denied the private
respondents petition to cancel the Certificate of
Candidacy (COC) of petitioner Abraham Kahlil B.
Mitra (Mitra).
In its Motion for Reconsideration dated July 19,
2010, the COMELEC, through the Office of the
Solicitor General, asks us to reconsider our July 2,
2010 Decision. The COMELEC argues that we
overstepped our review power over its factual
findings; as a specialized constitutional body, the
findings and conclusions of the COMELEC are

generally respected and even given the status of


finality.The COMELEC also contends that the
Court erred in taking cognizance of the present
petition since the issues raised therein are
essentially factual in nature.It claims that it is
elementary that the extraordinary remedy
ofcertiorariis limited to correcting questions of
law and that the factual issues raised in the
present petition are not appropriate for a petition
for review on certiorari.
ISSUES: Whether the SC erred when it reviewed
the probative value of the evidence presented
and substituted its own factual findings over that
of the public respondent.
Ruling:
POLITICAL LAW: judicial power
The COMELEC should likewise be aware that the
Constitution itself, in defining judicial power,
pointedly states that
Judicial power includes the duty of the courts of
justice to settle actual controversies involving
rights which are legally demandable and
enforceable, and to determine whether or not
there has been a grave abuse of discretion
amounting to lack or excess of jurisdiction on the
part of any branch or instrumentality of the
Government.
This provision, more than anything else, identifies
the power and duty of this Court in grave abuse
of discretion situations, and differentiates this
authority from the power of review by appeal that
Rule 45 of the Rules of Court defines.
Based on these considerations, we cannot accept
the COMELEC's position that patently confuses
the mode of review in election cases under Rules
64 and 65 of the Rules of Court, with the
appellate review that Rule 45 of the same Rules

provides.
We likewise reject the COMELEC and the private
respondents proposition that the Court erred in
exercising its limited certiorari
jurisdiction.Although the COMELEC is admittedly
the final arbiter of all factual issues as the
Constitution And the Rules of Court provide, we
stress that in the presence of grave abuse of
discretion, our constitutional duty is to intervene
and not to shy away from intervention simply
because a specialized agency has been given the
authority to resolve the factual issues.
As we emphasized in our Decision, we have in the
past recognized exceptions to the general rule
that the Court ordinarily does not review in
acertioraricase the COMELECs appreciation and
evaluation of evidence.One such exception is
when the COMELECs appreciation and evaluation
of evidence go beyond the limits of its discretion
to the point of being grossly unreasonable.In this
situation, we are duty bound under the
Constitution to intervene and correct COMELEC
errors that, because of the attendant grave abuse
of discretion, have mutated into errors of
jurisdiction.
Our Decision clearly pointed out Mitras
submissions and arguments on grave abuse of
discretion, namely, that the COMELEC failed to
appreciate that the case is a cancellation of a
COC proceeding and that the critical issue is the
presence of deliberate false material
representation to deceive the electorate.In fact,
Mitras petition plainly argued that the COMELECs
grave abuse of discretion was patent when it
failed to consider that the ground to deny a COC
is deliberate false representation.We completely
addressed this issue and, in the process,
analyzed the reasoning in the assailed COMELEC
decision. At every step, we found that the

COMELEC committed grave abuse of discretion in


the appreciation of the evidence.
Under these terms, we cannot be any clearer.
WHEREFORE, premises considered, we resolve to
DENY with FINALITY, for lack of merit, the motions
for reconsideration and motion for oral
arguments now before us.Let entry of judgment
be made in due course.

Salonga vs Cruz-Pano, 134 SCRA 438 (1985)


Facts: The petitioner invokes the constitutionally
protected right to life and liberty guaranteed by
the due process clause, alleging that no prima
facie case has been established to warrant the
filing of an information for subversion against
him. Petitioner asks the Court to prohibit and
prevent the respondents from using the iron arm
of the law to harass, oppress, and persecute him,
a member of the democratic opposition in the
Philippines.
The case roots backs to the rash of bombings
which occurred in the Metro Manila area in the
months of August, September and October of
1980. Victor Burns Lovely, Jr, one of the victims of
the bombing, implicated petitioner Salonga as
one of those responsible.
On December 10, 1980, the Judge Advocate
General sent the petitioner a Notice of
Preliminary Investigation in People v. Benigno
Aquino, Jr., et al. (which included petitioner as a
co-accused), stating that the preliminary
investigation of the above-entitled case has been
set at 2:30 oclock p.m. on December 12, 1980
and that petitioner was given ten (10) days from
receipt of the charge sheet and the supporting
evidence within which to file his counterevidence. The petitioner states that up to the

time martial law was lifted on January 17, 1981,


and despite assurance to the contrary, he has not
received any copies of the charges against him
nor any copies of the so-called supporting
evidence.
The counsel for Salonga was furnished a copy of
an amended complaint signed by Gen. Prospero
Olivas, dated 12 March 1981, charging Salonga,
along with 39 other accused with the violation of
RA 1700, as amended by PD 885, BP 31 and PD
1736. On 15 October 1981, the counsel for
Salonga filed a motion to dismiss the charges
against Salonga for failure of the prosecution to
establish a prima facie case against him. On 2
December 1981, Judge Ernani Cruz Pano
(Presiding Judge of the Court of First Instance of
Rizal, Branch XVIII, Quezon City) denied the
motion. On 4 January 1982, he (Pano) issued a
resolution ordering the filing of an information for
violation of the Revised Anti-Subversion Act, as
amended, against 40 people, including Salonga.
The resolutions of the said judge dated 2
December 1981 and 4 January 1982 are the
subject of the present petition for certiorari. It is
the contention of Salonga that no prima facie
case has been established by the prosecution to
justify the filing of an information against him. He
states that to sanction his further prosecution
despite the lack of evidence against him would be
to admit that no rule of law exists in the
Philippines today.
Issues: 1. Whether the above case still falls under
an actual case
2. Whether the above case dropped by the lower
court still deserves a decision from the Supreme
Court
Held: 1. No. The Court had already deliberated
on this case, a consensus on the Courts
judgment had been arrived at, and a draft

ponencia was circulating for concurrences and


separate opinions, if any, when on January 18,
1985, respondent Judge Rodolfo Ortiz granted the
motion of respondent City Fiscal Sergio Apostol
to drop the subversion case against the
petitioner. Pursuant to instructions of the
Minister of Justice, the prosecution restudied its
evidence and decided to seek the exclusion of
petitioner Jovito Salonga as one of the accused in
the information filed under the questioned
resolution.
The court is constrained by this action of the
prosecution and the respondent Judge to
withdraw the draft ponencia from circulating for
concurrences and signatures and to place it once
again in the Courts crowded agenda for further
deliberations.
Insofar as the absence of a prima facie case to
warrant the filing of subversion charges is
concerned, this decision has been rendered moot
and academic by the action of the prosecution.
2. Yes. Despite the SCs dismissal of the petition
due to the cases moot and academic nature, it
has on several occasions rendered elaborate
decisions in similar cases where mootness was
clearly apparent.
The Court also has the duty to formulate guiding
and controlling constitutional principles,
precepts, doctrines, or rules. It has the symbolic
function of educating bench and bar on the
extent of protection given by constitutional
guarantees.

EVELIO B. JAVIER, petitioner,


vs.
THE COMMISSION ON ELECTIONS, and ARTURO F.

PACIFICADOR, respondents.
Raul S. Roco and Lorna Patajo-Kapunan for petitioner.
Facts:
The petitioner and the private respondent were candidates in
Antique for the Batasang Pambansa in the May 1984 elections.
On May 13, 1984, the eve of the elections, the bitter contest
between the two came to a head when several followers of the
petitioner were ambushed and killed, allegedly by the latters
men. Seven suspects, including respondent Pacificador, are
now facing trial for these murders.
It was in this atmosphere that the voting was held, and the
post-election developments were to run true to form. Owing to
what he claimed were attempts to railroad the private
respondents proclamation, the petitioner went to the
Commission on Elections to question the canvass of the
election returns. His complaints were dismissed and the
private respondent was proclaimed winner by the Second
Division of the said body. The petitioner thereupon came to
this Court, arguing that the proclamation was void because
made only by a division and not by the Commission on
Elections en banc as required by the Constitution.
On May 18, 1984, the Second Division of the Commission on
Elections directed the provincial board of canvassers of
Antique to proceed with the canvass but to suspend the
proclamation of the winning candidate until further orders. On
June 7, 1984, the same Second Division ordered the board to
immediately convene and to proclaim the winner without
prejudice to the outcome of the case before the Commission.
On certiorari before this Court, the proclamation made by the
board of canvassers was set aside as premature, having been
made before the lapse of the 5-day period of appeal, which the
petitioner had seasonably made. Finally, on July 23, 1984, the
Second Division promulgated the decision now subject of this
petition which inter alia proclaimed Arturo F. Pacificador the
elected assemblyman of the province of Antique. The
petitioner then came to this Court, asking to annul the said
decision on the basis that it should have been decided by
COMELEC en banc.
The case was still being considered when on February 11,
1986, the petitioner was gunned down in cold blood and in
broad daylight. And a year later, Batasang Pambansa was
abolished with the advent of the 1987 Constitution.

Respondents moved to dismiss the petition, contending it to


be moot and academic.
Issues:
1. Whether it is correct for the court to dismiss the petition
due to the petitioner being dead and the respondent missing.
2. Whether the Second Division of the Commission on
Elections was authorized to promulgate its decision of July 23,
1984, proclaiming the private respondent the winner in the
election?
Held:
1. No.
The abolition of the Batasang Pambansa and the
disappearance of the office in dispute between the petitioner
and the private respondent-both of whom have gone their
separate ways-could be a convenient justification for
dismissing this case. But there are larger issues involved that
must be resolved now, once and for all, not only to dispel the
legal ambiguities here raised. The more important purpose is
to manifest in the clearest possible terms that this Court will
not disregard and in effect condone wrong on the simplistic
and tolerant pretext that the case has become moot and
academic.
The Supreme Court is not only the highest arbiter of legal
questions but also the conscience of the government. The
citizen comes to us in quest of law but we must also give him
justice. The two are not always the same. There are times
when we cannot grant the latter because the issue has been
settled and decision is no longer possible according to the law.
But there are also times when although the dispute has
disappeared, as in this case, it nevertheless cries out to be
resolved. Justice demands that we act then, not only for the
vindication of the outraged right, though gone, but also for the
guidance of and as a restraint upon the future.
2. No.
The applicable provisions are found in Article XII-C, Sections 2
and 3, of the 1973 Constitution.
Section 2 confers on the Commission on Elections the power
to:
(2) Be the sole judge of all contests relating to the election,
returns and qualifications of all member of the Batasang
Pambansa and elective provincial and city officials.
Section 3 provides:
The Commission on Elections may sit en banc or in three

divisions. All election cases may be heard and decided by


divisions except contests involving members of the Batasang
Pambansa, which shall be heard and decided en banc. Unless
otherwise provided by law, all election cases shall be decided
within ninety days from the date of their submission for
decision.
We believe that in making the Commission on Elections the
sole judge of all contests involving the election, returns and
qualifications of the members of the Batasang Pambansa and
elective provincial and city officials, the Constitution intended
to give it full authority to hear and decide these cases from
beginning to end and on all matters related thereto, including
those arising before the proclamation of the winners.
As correctly observed by the petitioner, the purpose of Section
3 in requiring that cases involving members of the Batasang
Pambansa be heard and decided by the Commission en banc
was to insure the most careful consideration of such cases.
Obviously, that objective could not be achieved if the
Commission could act en banc only after the proclamation had
been made, for it might then be too late already. We are alltoo-familiar with the grab-the-proclamation-and-delay-theprotest strategy of many unscrupulous candidates, which has
resulted in the frustration of the popular will and the virtual
defeat of the real winners in the election. The respondents
theory would make this gambit possible for the preproclamation proceedings, being summary in nature, could be
hastily decided by only three members in division, without the
care and deliberation that would have otherwise been
observed by the Commission en banc.
WHEREFORE, let it be spread in the records of this case that
were it not for the supervening events that have legally
rendered it moot and academic, this petition would have been
granted and the decision of the Commission on Elections
dated July 23, 1984, set aside as violative of the Constitution.
Ynot v IAC (1987) 148 SCRA 659
J. Cruz
Facts:
Petitioner transported 6 caracbaos from Masbate to Iloilo in
1984 and these wer confiscated by the station commander in
Barotac, Iloilo for violating E.O. 626 A which prohibits

transportation of a carabao or carabeef from one province to


another. Confiscation will be a result of this.
The petitioner sued for recovery, and the Regional Trial Court
of Iloilo City issued a writ of replevin upon his filing of a
supersedeas bond of P12,000.00. After considering the merits
of the case, the court sustained the confiscation of the
carabaos and, since they could no longer be produced,
ordered the confiscation of the bond. The court also declined
to rule on the constitutionality of the executive order, as raise
by the petitioner, for lack of authority and also for its
presumed validity.
The same result was decided in the trial court.
In the Supreme Court, he then petitioned against the
constitutionality of the E.O. due to the outright confiscation
without giving the owner the right to heard before an impartial
court as guaranteed by due process. He also challenged the
improper exercise of legislative power by the former president
under Amendment 6 of the 1973 constitution wherein Marcos
was given emergency powers to issue letters of instruction
that had the force of law.
Issue: Is the E.O. constitutional?
Holding: The EO is unconstitutional. Petition granted.
Ratio:
The lower courts are not prevented from examining the
constitutionality of a law.
Constitutional grant to the supreme court to review.
Justice Laurel's said, courts should not follow the path of least
resistance by simply presuming the constitutionality of a law
when it is questioned. On the contrary, they should probe the
issue more deeply, to relieve the abscess, and so heal the
wound or excise the affliction.
The challenged measure is denominated an executive order
but it is really presidential decree, promulgating a new rule
instead of merely implementing an existing law due to the
grant of legislative authority over the president under
Amendment number 6.
Provisions of the constitution should be cast in precise
language to avoid controvery. In the due process clause,
however, the wording was ambiguous so it would remain
resilient. This was due to the avoidance of an iron rule laying

down a stiff command for all circumstances. There was


flexibility to allow it to adapt to every situation with varying
degrees at protection for the changing conditions.
Courts have also refrained to adopt a standard definition for
due processlest they be confined to its interpretation like a
straitjacket.
There must be requirements of notice and hearing as a
safeguard against arbitrariness.
There are exceptions such as conclusive presumption which
bars omission of contrary evidence as long as such
presumption is based on human experience or rational
connection between facts proved and fact presumed. An
examples is a passport of a person with a criminal offense
cancelled without hearing.
The protection of the general welfare is the particular function
of police power which both restrains and is restrained by dure
process. This power was invoked in 626-A, in addition to 626
which prohibits slaughter of carabos with an exception.
While 626-A has the same lawful subjectas the original
executive order, it cant be said that it complies with the
existence of a lawful method. The transport prohibition and
the purpose sought has a gap.
Summary action may be taken in valid admin proceedings as
procedural due process is not juridical only due to the urgency
needed to correct it.
There was no reason why the offense in the E.O. would not
have been proved in a court of justice with the accused
acquired the rights in the constitution.
The challenged measure was an invalid exercise of police
power because the method toconfiscate carabos was
oppressive.
Due process was violated because the owener was denied the
right to be heard or his defense and punished immediately.
This was a clear encroachment on judicial functions and
against the separataion of powers.
The policeman wasnt liable for damages since the law during
that time was valid.
Serrano de Agbayani vs PNB, 38 SCRA 429 (1972)

Agbayani obtained a loan P450 from PNB secured by a


REM, which was to mature 5 years later.
15 years later, PNB sought to foreclose the REM.
Agbayani filed a complaint claiming that it was barred by
prescription. She also claims that she obtained an
injunction against the sheriff.
PNB argued that the claim has not yet prescribed if the
period from the time of issuance of EO32 to the time when
RA 342 was issued should be deducted.
o E0 32 was issued in 1945 providing for debt
moratorium
o RA 342 was issued in 1948 - extension of the debt
moratorium
The RA 342 was declared void and since it was an
extension of EO 32, EO 32 was likewise nullified.
Here, RA 342 (the debt moratorium law) continued EO 32,
suspending the payment of debts by war sufferers.
However RA 342 could not pass the test of validity. (I think
what Justice Fernando was saying is that the law was later
declared unconstitutional because it violates the nonimpairment of contractual obligations clause in the
constitution).
PNB claims that this period should be deducted from the
prescriptive period since during this time the bank took no
legal steps for the recovery of the loan. As such, the action
has not yet prescribed.
ISSUE: Has the action prescribed?
HELD:
The general rule is that an unconstitutional act because it
suffers from infirmity, cannot be a source of legal rights or
duties. When the courts declare a law to be inconsistent with
the Constitution, the former shall be void and the latter shall
govern.
However, prior to the declaration of nullity of such challenged
legislative act must have been in force and had to be complied
with. This is so as until after the judiciary, in an appropriate
case declares its invalidity, it is entitled to obedience and
respect. Such legislative act was in operation and presumed to
be valid in all respects. It is now accepted that prior to its
being nullified, its existence as a fact must be reckoned with.
This is merely to reflect the awareness that precisely because
the judiciary is the governmental organ which has the final say

on whether a legislative act is valid, a period of time may have


elapsed before it can exercise the power of judicial review that
may lead to a declaration of nullity. It would e to deprive the
law of its quality of fairness and justice then, if there be no
recognition of what had transpired prior to such adjudication.
The past cannot always be erased by judicial declaration.
(OPERATIVE FACT DOCTRINE). The existence of a statute prior
to its being adjudged void is an operative fact to which legal
consequences are attached.
During the 8 year period that EO 32 and RA 342 were in force,
prescription did not run. Thus, the prescriptive period was
tolled in the meantime prior to such adjudication of invalidity.

SALAZAR VS. ACHACOSO [183 SCRA 145 ]


Facts: Rosalie Tesoro of Pasay City in a sworn statement filed
with the POEA, charged petitioner with illegal recruitment.
Public respondent Atty. Ferdinand Marquez sent petitioner a
telegram directing him to appear to the POEA regarding the
complaint against him. On the same day, after knowing that
petitioner had no license to operate a recruitment agency,
public respondent Administrator Tomas Achacoso issued a
Closure and Seizure Order No. 1205 to petitioner. It stated that
there will a seizure of the documents and paraphernalia being
used or intended to be used as the means of committing
illegal recruitment, it having verified that petitioner has (1)
No valid license or authority from the Department of Labor and
Employment to recruit and deploy workers for overseas
employment; (2) Committed/are committing acts prohibited
under Article 34 of the New Labor Code in relation to Article 38
of the same code. A team was then tasked to implement the
said Order. The group, accompanied by mediamen and
Mandaluyong policemen, went to petitioners residence. They
served the order to a certain Mrs. For a Salazar, who let them
in. The team confiscated assorted costumes. Petitioner filed
with POEA a letter requesting for the return of the seized
properties, because she was not given prior notice and
hearing. The said Order violated due process. She also alleged
that it violated sec 2 of the Bill of Rights, and the properties
were confiscated against her will and were done with

unreasonable force and intimidation.


Issue: Whether or Not the Philippine Overseas Employment
Administration (or the Secretary of Labor) can validly issue
warrants of search and seizure (or arrest) under Article 38 of
the Labor Code
Held: Under the new Constitution, . . . no search warrant or
warrant of arrest shall issue except upon probable cause to be
determined personally by the judge after examination under
oath or affirmation of the complainant and the witnesses he
may produce, and particularly describing the place to be
searched and the persons or things to be seized. Mayors and
prosecuting officers cannot issue warrants of seizure or arrest.
The Closure and Seizure Order was based on Article 38 of the
Labor Code. The Supreme Court held, We reiterate that the
Secretary of Labor, not being a judge, may no longer issue
search or arrest warrants. Hence, the authorities must go
through the judicial process. To that extent, we declare Article
38, paragraph (c), of the Labor Code, unconstitutional and of
no force and effect The power of the President to order the
arrest of aliens for deportation is, obviously, exceptional. It
(the power to order arrests) cannot be made to extend to
other cases, like the one at bar. Under the Constitution, it is
the sole domain of the courts. Furthermore, the search and
seizure order was in the nature of a general warrant. The court
held that the warrant is null and void, because it must identify
specifically the things to be seized.
WHEREFORE, the petition is GRANTED. Article 38, paragraph
(c) of the Labor Code is declared UNCONSTITUTIONAL and null
and void. The respondents are ORDERED to return all
materials seized as a result of the implementation of Search
and Seizure Order No. 1205.
VIII. CITIZENSHIP

Lee vs. Director of Lands


G.R. No. 128195
October 3, 2001
FACTS:
Sometime in March 1936, the Dinglasans sold to Lee
Liong (Chinese citizen) a parcel of land situated at the
corner of Roxas Avenue and Pavia Street, Roxas City.
In 1993, Elizabeth Manuel-Lee and Pacita Yu Lee filed
with the RTC of Roxas City a petition for reconstitution of
title of the lot. (Alleging that the transfer certificate of
title issued to Lee Liong was lost or destroyed during
World War II.)
Petitioners Elizabeth and Pacita alleged that they were
the widows of the deceased Lee Bing Hoo and Lee
Bun Ting, who were the heirs of Lee Liong, the owner
of the lot.
The RTC approved reconstitution of the lost or destroyed
certificate of title in the name of Lee Liong on the basis
of an approved plan and technical description.
Solicitor General filed with the Court of Appeals a
petition for annulment of the RTC decision alleging
that the RTC had no jurisdiction over the case.
The Solicitor General contended that the petitioners
were not the proper parties in the reconstitution of title,
since their predecessor-in-interest Lee Liong did not
acquire title to the lot because he was a Chinese citizen
and was constitutionally not qualified to own the subject
land.
CA declared the reconstitution void. Hence this petition.
Elizabeth and Pacita emphasized that the ownership of
the land had been settled in two previous cases of the
Supreme Court, where the Court ruled in favor of their
predecessor-in-interest, Lee Liong.
They also pointed out that they acquired ownership of
the land through actual possession of the lot and their
consistent payment of taxes over the land for more than
sixty years.
On the other hand, the Solicitor General submitted
that the decision in the reconstitution case was void;
otherwise, it would amount to circumventing the

constitutional proscription against aliens acquiring


ownership of private or public agricultural lands.
ISSUES:
1. WON Lee Liong has the qualification to own land in the
Philippines. NO
2. WON the reconstitution was valid. NO
HELD:
1. Lee Liong was not qualified but the ownership of
the lot was already acquired by Filipino citizens Lee
Liong was disqualified to acquire the land under the
1935 Constitution. The sale of the land in question
was consummated sometime in March 1936, during
the effectivity of the 1935 Constitution.

from falling in the hands of non-Filipinos. In this case, however,


there would be no more public policy violated since the land is
in the hands of Filipinos qualified to acquire and own such
land.
If land is invalidly transferred to an alien who subsequently
becomes a citizen or transfers it to a citizen, the flaw in the
original transaction is considered cured and the title of the
transferee is rendered valid.
Thus, the subsequent transfer of the property to qualified
Filipinos may no longer be impugned on the basis of the
invalidity of the initial transfer. The objective of the
constitutional provision to keep our lands in Filipino hands has
been achieved.

Under the 1935 Constitution aliens could not acquire private


agricultural lands, save in cases of hereditary succession.
Thus, Lee Liong, a Chinese citizen, was disqualified to acquire
the land in question.

Incidentally, it must be mentioned that reconstitution of the


original certificate of title must be based on an owners
duplicate, secondary evidence thereof, or other valid sources
of the title to be reconstituted.

The fact that the Court did not annul the sale of the land to an
alien did not validate the transaction. It was still contrary to
the constitutional proscription against aliens acquiring lands of
the public or private domain.

2. Reconstitution was void for lack of factual support


In this case, reconstitution was based on the plan and
technical description approved by the Land Registration
Authority. This renders the order of reconstitution void for lack
of factual support. A judgment with absolutely nothing to
support it is void.

The proper party to assail the sale is the Solicitor General.


This was what was done in this case when the Solicitor
General initiated an action for annulment of judgment of
reconstitution of title. While it took the Republic more than
sixty years to assert itself, it is not barred from initiating such
action. Prescription never lies against the State.
The land is now in the hands of Filipinos.
The original vendee, Lee Liong, has since died and the land
has been inherited by his heirs and subsequently their heirs,
petitioners herein. Petitioners are Filipino citizens, a fact the
Solicitor General does not dispute.
The constitutional proscription on alien ownership of lands of
the public or private domain was intended to protect lands

As earlier mentioned, a reconstitution of title is the reissuance of a new certificate of title lost or destroyed in its
original form and condition. It does not pass upon the
ownership of the land covered by the lost or destroyed title.
Any change in the ownership of the property must be the
subject of a separate suit. Thus, although petitioners are in
possession of the land, a separate proceeding is necessary to
thresh out the issue of ownership of the land.
The SC Court REVERSES and SETS ASIDE the decision of the
CA.

G.R. No. 153883 January 13, 2004


Republic of the Philippines v Chule Y Lim
Facts:
The respondent, Chule Y. Lim, is an illegitimate daughter of a
Chinese father and a Filipina mother, who never got married
due to a prior subsisting marriage of her father. The
respondent petitioned that there were few mistakes as to her
citizenship and identity, to wit:
1. That her surname Yu was misspelled as Yo. She has
been using Yu in all of her school records and in her
marriage certificate.
2. That her fathers name in her birth record was written as Yo
Diu To (Co Tian) when it should have been Yu Dio To (Co
Tian).
3. That her nationality was entered as Chinese when it should
have been Filipino considering that her father and mother got
married.
4. That she was entered as a legitimate child on her birth
certificate when in fact, it should have been illegitimate. Both
the trial court and Court of Appeals granted the respondents
petition.
Issue:
The Republic of the Philippines appealed the decision to the
Supreme Court on the following grounds:
1. Whether the Court of Appeals erred in ordering the
correction of the citizenship of respondent Chule Y. Lim from
Chinese to Filipino despite the fact that respondent never
demonstrated any compliance with the legal requirements for
election of citizenship.
2. Whether the Court of Appeals erred in allowing respondent
to continue using her fathers surname despite its finding that
respondent is an illegitimate child.
Held:
1. No. The Republic avers that respondent did not comply with
the constitutional requirement of electing Filipino citizenship
when she reached the age of majority as mandated in Article
IV, Section 1(3) of the 1935 Constitution and Section 1 of the
Commonwealth Act No. 625. The Supreme Court held that the
two above provisions only apply to legitimate children. These
do not apply in the case of the respondent who was an
illegitimate child considering that her parents never got

married. By being an illegitimate child of a Filipino mother,


respondent automatically became a Filipino upon birth, and as
such, there was no more need for her to validly elect Filipino
citizenship upon reaching the age of majority. Also, she
registered as a voter inside the country when she reached 18
years old. The exercise of the right of suffrage and the
participation in election exercises constitute a positive act of
election of Philippine citizenship.
2. No. The Republics submission was misleading. The Court
of Appeals did not allow respondent to use her fathers
surname. What it did allow was the correction of her fathers
misspelled surname which she has been using ever since she
can remember. The court held that prohibiting the respondent
to use her fathers surname would only sow confusion. Also,
Sec. 1 of Commonwealth Act No. 142 which regulates the use
of aliases as well as the jurisprudence state that it is allowed
for a person to use a name by which he has been known
since childhood. Even legitimate children cannot enjoin the
illegitimate children of their father from using his surname.
While judicial authority is required for a chance of name or
surname, there is no such requirement for the continued use
of a surname which a person has already been using since
childhood.
The doctrine that disallows such change of name as would
give the false impression of family relationship remains valid
but only to the extent that the proposed change of name
would in great probability cause prejudice or future mischief to
the family whose surname it is that is involved or to the
community in general. In this case, the Republic has not
shown that the Yu family in China would probably be
prejudiced or be the object of future mischief.
WHEREFORE, in view of the foregoing, the instant petition
brought by the Republic is DENIED. The decision of the Court
of Appeals is AFFIRMED.

Tecson vs. Commission on Elections


[GR 151434, 3 March 2004]
Facts: On 31 December 2003, Ronald Allan Kelly Poe, also
known as Fernando Poe, Jr. (FPJ), filed his certificate of
candidacy for the position of President of the Republic of the

Philippines under the Koalisyon ng Nagkakaisang Pilipino (KNP)


Party, in the 2004 national elections. In his certificate of
candidacy, FPJ, representing himself to be a natural-born
citizen of the Philippines, stated his name to be "Fernando Jr.,"
or "Ronald Allan" Poe, his date of birth to be 20 August 1939
and his place of birth to be Manila. Victorino X. Fornier, (GR
161824) initiated, on 9 January 2004, a petition (SPA 04-003)
before the Commission on Elections (COMELEC) to disqualify
FPJ and to deny due course or to cancel his certificate of
candidacy upon the thesis that FPJ made a material
misrepresentation in his certificate of candidacy by claiming to
be a natural-born Filipino citizen when in truth, according to
Fornier, his parents were foreigners; his mother, Bessie Kelley
Poe, was an American, and his father, Allan Poe, was a Spanish
national, being the son of Lorenzo Pou, a Spanish subject.
Granting, Fornier asseverated, that Allan F. Poe was a Filipino
citizen, he could not have transmitted his Filipino citizenship to
FPJ, the latter being an illegitimate child of an alien mother.
Fornier based the allegation of the illegitimate birth of FPJ on
two assertions: (1) Allan F. Poe contracted a prior marriage to a
certain Paulita Gomez before his marriage to Bessie Kelley
and, (2) even if no such prior marriage had existed, Allan F.
Poe, married Bessie Kelly only a year after the birth of FPJ. On
23 January 2004, the COMELEC dismissed SPA 04-003 for lack
of merit. 3 days later, or on 26 January 2004, Fornier filed his
motion for reconsideration. The motion was denied on 6
February 2004 by the COMELEC en banc. On 10 February
2004, Fornier assailed the decision of the COMELEC before the
Supreme Court conformably with Rule 64, in relation to Rule
65, of the Revised Rules of Civil Procedure. The petition
likewise prayed for a temporary restraining order, a writ of
preliminary injunction or any other resolution that would stay
the finality and/or execution of the COMELEC resolutions. The
other petitions, later consolidated with GR 161824, would
include GR 161434 and GR 161634, both challenging the
jurisdiction of the COMELEC and asserting that, under Article
VII, Section 4, paragraph 7, of the 1987 Constitution, only the
Supreme Court had original and exclusive jurisdiction to
resolve the basic issue on the case.
Issue: Whether FPJ was a natural born citizen, so as to be
allowed to run for the offcie of the President of the Philippines.

Held: Section 2, Article VII, of the 1987 Constitution expresses


that "No person may be elected President unless he is a
natural-born citizen of the Philippines, a registered voter, able
to read and write, at least forty years of age on the day of the
election, and a resident of the Philippines for at least ten years
immediately preceding such election." The term "natural-born
citizens," is defined to include "those who are citizens of the
Philippines from birth without having to perform any act to
acquire or perfect their Philippine citizenship." Herein, the
date, month and year of birth of FPJ appeared to be 20 August
1939 during the regime of the 1935 Constitution. Through its
history, four modes of acquiring citizenship - naturalization, jus
soli, res judicata and jus sanguinis had been in vogue. Only
two, i.e., jus soli and jus sanguinis, could qualify a person to
being a natural-born citizen of the Philippines. Jus soli, per
Roa vs. Collector of Customs (1912), did not last long. With the
adoption of the 1935 Constitution and the reversal of Roa in
Tan Chong vs. Secretary of Labor (1947), jus sanguinis or blood
relationship would now become the primary basis of
citizenship by birth. Considering the reservations made by the
parties on the veracity of some of the entries on the birth
certificate of FPJ and the marriage certificate of his parents,
the only conclusions that could be drawn with some degree of
certainty from the documents would be that (1) The parents of
FPJ were Allan F. Poe and Bessie Kelley; (2) FPJ was born to
them on 20 August 1939; (3) Allan F. Poe and Bessie Kelley
were married to each other on 16 September, 1940; (4) The
father of Allan F. Poe was Lorenzo Poe; and (5) At the time of
his death on 11 September 1954, Lorenzo Poe was 84 years
old. The marriage certificate of Allan F. Poe and Bessie Kelley,
the birth certificate of FPJ, and the death certificate of Lorenzo
Pou are documents of public record in the custody of a public
officer. The documents have been submitted in evidence by
both contending parties during the proceedings before the
COMELEC. But while the totality of the evidence may not
establish conclusively that FPJ is a natural-born citizen of the
Philippines, the evidence on hand still would preponderate in
his favor enough to hold that he cannot be held guilty of
having made a material misrepresentation in his certificate of
candidacy in violation of Section 78, in relation to Section 74,
of the Omnibus Election Code. Fornier has utterly failed to
substantiate his case before the Court, notwithstanding the
ample opportunity given to the parties to present their

position and evidence, and to prove whether or not there has


been material misrepresentation, which, as so ruled in
Romualdez-Marcos vs. COMELEC, must not only be material,
but also deliberate and willful. The petitions were dismissed.

In re Ching, Bar Matter No. 914, October 1, 1999

Bar Matter No. 914, October 1, 1999


Re: Application for Admission to the Philippine Bar
vs.
Vicente D. Ching, petitioner
Facts:
Vicente D. Ching, a legitimate child of a Filipino mother and an
alien Chinese father, was born on April 11, 1964 in Tubao La
Union, under the 1935 Constitution. He has resided in the
Philippines
He completed his Bachelor of Laws at SLU in Baguio on July
1998, filed an application to take the 1998 Bar Examination.
The Resolution in this Court, he was allowed to take the bar if
he submit to the Court the following documents as proof of his
Philippine Citizenship:
1. Certification issued by the PRC Board of Accountancy that
Ching is a certified accountant;
2. Voter Certification issued COMELEC in Tubao La Union
showing that Ching is a registered voter of his place; and
3. Certification showing that Ching was elected as member of
the Sangguniang Bayan of Tubao, La Union
On April 5, 1999, Ching was one of the bar passers. The oath
taking ceremony was scheduled on May 5, 1999.
Because of his questionable status of Ching's citizenship, he
was not allowed to take oath.
He was required to submit further proof of his citizenship.
The Office of the Solicitor General was required to file a
comment on Ching's petition for admission to the Philippine
Bar.
In his report:
1. Ching, under the 1935 Constitution, was a Chinese citizen
and continue to be so, unless upon reaching the age of
majority he elected Philippine citizenship, under the

compliance with the provisions of Commonwealth Act No. 265


"an act providing for the manner in which the option to elect
Philippine citizenship shall be declared by a person whose
mother is a Filipino citizen"
2. He pointed out the Ching has not formally elected Philippine
citizenship, and if ever he does, it would already be beyond
the "reasonable time" allowed by the present jurisprudence.
Issue:
Whether or not he has elected Philippine citizenship within "a
reasonable time".
Rulings:
No. Ching, despite the special circumstances, failed to elect
Philippine citizenship within a reasonable time. The reasonable
time means that the election should be made within 3 years
from "upon reaching the age of majority", which is 21 years
old. Instead, he elected Philippine citizenship 14 years after
reaching the age of majority which the court considered not
within the reasonable time. Ching offered no reason why he
delayed his election of Philippine citizenship, as procedure in
electing Philippine citizenship is not a tedious and painstaking
process. All that is required is an affidavit of election of
Philippine citizenship and file the same with the nearest civil
registry.

Co vs HRET, 199 SCRA 692 (1991)

Co vs HRET, 199 SCRA 692 (1991)


Facts:
The HRET declared that respondent Jose Ong, Jr. is a natural
born Filipino citizen and a resident of Laoang, Northern Samar
for voting purposes. The congressional election for the second
district of Northern Samar was held. Among the candidates
who vied for the position of representative in the second
legislative district are the petitioners, Sixto Balinquit and
Antonio Co and the private respondent, Jose Ong, Jr.
Respondent Ong was proclaimed the duly elected
representative of the second district of Northern Samar.

The petitioners filed election protests on the grounds that Jose


Ong, Jr. is not a natural born citizen of the Philippines and not a
resident of the second district of Northern Samar.

BENGSON vs. HRET and CRUZ


G.R. No. 142840
May 7, 2001

Issue: Whether or not Jose Ong, Jr. is a citizen of the


Philippines.

FACTS: The citizenship of respondent Cruz is at issue in this


case, in view of the constitutional requirement that no person
shall be a Member of the House of Representatives unless he
is a natural-born citizen.
Cruz was a natural-born citizen of the Philippines. He was born
in Tarlac in 1960 of Filipino parents. In 1985, however, Cruz
enlisted in the US Marine Corps and without the consent of the
Republic of the Philippines, took an oath of allegiance to the
USA. As a Consequence, he lost his Filipino citizenship for
under CA No. 63 [(An Act Providing for the Ways in Which
Philippine Citizenship May Be Lost or Reacquired (1936)]
section 1(4), a Filipino citizen may lose his citizenship by,
among other, rendering service to or accepting commission in
the armed forces of a foreign country.
Whatever doubt that remained regarding his loss of Philippine
citizenship was erased by his naturalization as a U.S. citizen in
1990, in connection with his service in the U.S. Marine Corps.
In 1994, Cruz reacquired his Philippine citizenship through
repatriation under RA 2630 [(An Act Providing for Reacquisition
of Philippine Citizenship by Persons Who Lost Such Citizenship
by Rendering Service To, or Accepting Commission In, the
Armed Forces of the United States (1960)]. He ran for and was
elected as the Representative of the 2nd District of Pangasinan
in the 1998 elections. He won over petitioner Bengson who
was then running for reelection.
Subsequently, petitioner filed a case for Quo Warranto Ad
Cautelam with respondent HRET claiming that Cruz was not
qualified to become a member of the HOR since he is not a
natural-born citizen as required under Article VI, section 6 of
the Constitution.
HRET rendered its decision dismissing the petition for quo
warranto and declaring Cruz the duly elected Representative
in the said election.

Held:
Yes. In the year 1895, the private respondents grandfather,
Ong Te, arrived in the Philippines from China and established
his residence in the municipality of Laoang, Samar. The father
of the private respondent, Jose Ong Chuan was born in China
in 1905 but was brought by Ong Te to Samar in the year 1915,
he filed with the court an application for naturalization and
was declared a Filipino citizen.
In 1984, the private respondent married a Filipina named
Desiree Lim. For the elections of 1984 and 1986, Jose Ong, Jr.
registered himself as a voter of Laoang, Samar, and voted
there during those elections.
Under the 1973 Constitution, those born of Filipino fathers and
those born of Filipino mothers with an alien father were placed
on equal footing. They were both considered as natural born
citizens. Besides, private respondent did more than merely
exercise his right of suffrage. He has established his life here
in the Philippines.
On the issue of residence, it is not required that a person
should have a house in order to establish his residence and
domicile. It is enough that he should live in the municipality or
in a rented house or in that of a friend or relative. To require
him to own property in order to be eligible to run for Congress
would be tantamount to a property qualification. The
Constitution only requires that the candidate meet the age,
citizenship, voting and residence requirements.

Villahermosa vs Commissioner, 80 Phil 541 (1948)


Bengzon v. HRET, G.R. No. 142840, May 7, 2001

ISSUE: WON Cruz, a natural-born Filipino who became an


American citizen, can still be considered a natural-born Filipino
upon his reacquisition of Philippine citizenship.
HELD: petition dismissed

YES
Filipino citizens who have lost their citizenship may however
reacquire the same in the manner provided by law. C.A. No. 63
enumerates the 3 modes by which Philippine citizenship may
be reacquired by a former citizen:
1. by naturalization,
2. by repatriation, and
3. by direct act of Congress.
**
Repatriation may be had under various statutes by those who
lost their citizenship due to:
1. desertion of the armed forces;
2. services in the armed forces of the allied forces in World
War II;
3. service in the Armed Forces of the United States at any
other time,
4. marriage of a Filipino woman to an alien; and
5. political economic necessity
Repatriation results in the recovery of the original nationality
This means that a naturalized Filipino who lost his citizenship
will be restored to his prior status as a naturalized Filipino
citizen. On the other hand, if he was originally a natural-born
citizen before he lost his Philippine citizenship, he will be
restored to his former status as a natural-born Filipino.
R.A. No. 2630 provides:
Sec 1. Any person who had lost his Philippine citizenship by
rendering service to, or accepting commission in, the Armed
Forces of the United States, or after separation from the
Armed Forces of the United States, acquired United States
citizenship, may reacquire Philippine citizenship by taking an
oath of allegiance to the Republic of the Philippines and
registering the same with Local Civil Registry in the place
where he resides or last resided in the Philippines. The said
oath of allegiance shall contain a renunciation of any other
citizenship.
Having thus taken the required oath of allegiance to the
Republic and having registered the same in the Civil Registry
of Magantarem, Pangasinan in accordance with the aforecited
provision, Cruz is deemed to have recovered his original status
as a natural-born citizen, a status which he acquired at birth as
the son of a Filipino father. It bears stressing that the act of

repatriation allows him to recover, or return to, his original


status before he lost his Philippine citizenship.

Republic vs De La Rosa, G.R. No. 104654, June 6,


1994

REPUBLIC VS DELA ROSA


G.R. No. 104654, 6 June 1994 [Citizenship; Naturalization;
Naturalization Proceedings; C.A. No. 473]
FACTS:
September 20, 1991 - Frivaldo filed a petition for naturalization
under the Commonwealth Act No. 63 before the RTC Manila.
October 7, 1991 - Judge dela Rosa set the petition for hearing
on March 16, 1992, and directed the publication of the said
order and petition in the Official Gazette and a newspaper of
general circulation, for 3 consecutive weeks, the last
publication of which should be at least 6 months before the
date of the said hearing.
January 14, 1992 - Frivaldo asked the Judge to cancel the
March 16 hearing and move it to January 24, 1992, citing his
intention to run for public office in the May 1992 elections.
Judge granted the motion and the hearing was moved to
February 21. No publication or copy was issued about the
order.
February 21, 1992 - the hearing proceeded.
February 27, 1992 - Judge rendered the assailed Decision and
held that Frivaldo is readmitted as a citizen of the Republic of
the Philippines by naturalization.
Republic of the Philippines filed a petition for Certiorari
under Rule 45 of the Revised Rules of Court in relation to R.A.
No. 5440 and Section 25 of the Interim Rules, to annul the
decision made on February 27, 1992 and to nullify the oath of
allegiance taken by Frivaldo on same date.
ISSUE:
Whether or not Frivaldo was duly re-admitted to his citizenship
as a Filipino.

RULING:
No. The supreme court ruled that Private respondent is
declared NOT a citizen of the Philippines and therefore
disqualified from continuing to serve as governor of the
Province of Sorsogon. He is ordered to vacate his office and to
surrender the same to the Vice-Governor of the Province of
Sorsogon once this decision becomes final and executory. The
proceedings of the trial court was marred by the following
irregularities:
(1) the hearing of the petition was set ahead of the scheduled
date of hearing, without a publication of the order advancing
the date of hearing, and the petition itself;
(2) the petition was heard within six months from the last
publication of the petition;
(3) petitioner was allowed to take his oath of allegiance before
the finality of the judgment; and
(4) petitioner took his oath of allegiance without observing the
two-year waiting period.

Republic vs. Liyao, 214 SCRA 748 (1992)


G.R. No. L-35947, 20 October 1992 [Citizenship;
Naturalization; Grounds for Denaturalization]
FACTS:
William Li Yao was naturalized as a Filipino citizen pursuant to
the C.A. No. 473 and R.A. No. 530. 15 years later, the Solicitor
General filed a petition to cancel his naturalization because he
obtained such through fraud and tax evasion. Li Yao denied
the allegations.
ISSUE:
Whether or not the cancellation of Li Yao's naturalization is
valid.
RULING:
Yes. The cancellation of the naturalization certificate of Li Yao
was on the ground that it was "fraudulently and illegally
obtained" based on the Section 18(a) of CA 473, also known as
the Revised Naturalization Law.

A certificate of naturalization may be cancelled if it is


subsequently discovered that the applicant obtained it by
misleading the court upon any material fact. Law and
jurisprudence even authorize the cancellation of a certificate
of naturalization upon ground had conditions arising
subsequent to the granting of the certificate. Moreover, a
naturalization proceeding is not a judicial adversary
proceeding, the decision rendered therein, not constituting res
judicata as to any matter that would support a judgment
cancelling a certificate of naturalization on the ground of
illegal or fraudulent procurement thereof.

Moya Lim Yao v. Commissioner, 41 SCRA 292

MOY YA LIM YAO VS. COMMISSIONER OF IMMIGRATION


G.R. No. L-21289, October 4 1971, 41 SCRA 292
FACTS:
Lau Yuen Yeung applied for a passport visa to enter the
Philippines as a non-immigrant on 8 February 1961. In the
interrogation made in connection with herapplication for a
temporary visitor's visa to enter the Philippines, she stated
that she was a Chinese residing at Kowloon, Hongkong, and
that she desired to take a pleasure trip to the Philippines
to visit her great grand uncle, Lau Ching Ping. She was
permitted to come into the Philippines on 13 March 1961 for a
period of one month.
On the date of her arrival, Asher Y. Cheng filed a bond in the
amount of P1,000.00 to undertake, among others, that said
Lau Yuen Yeung would actually depart from the Philippines on
or before the expiration of her authorized period of stay in this
country or within the period as in his discretion the
Commissioner of Immigration or his authorized representative
might properly allow.
After repeated extensions, Lau Yuen Yeung was allowed to stay
in the Philippines up to 13 February 1962. On 25 January 1962,
she contracted marriage with Moy Ya Lim Yao alias Edilberto
Aguinaldo Lim an alleged Filipino citizen. Because of the
contemplated action of the Commissioner of Immigration to
confiscate her bond and order her arrest and immediate

deportation, after the expiration of her authorized stay, she


brought an action for injunction. At the hearing which took
place one and a half years after her arrival, it was admitted
that Lau Yuen Yeung could not write and speak
either English or Tagalog, except for a few words. She could
not name any Filipino neighbor, with a Filipino name except
one, Rosa. She did not know the names of her brothers-in-law,
or sisters-in-law. As a result, the Court of First Instance of
Manila denied the prayer for preliminary injunction. Moya Lim
Yao and Lau Yuen Yeung appealed.
ISSUE:
Whether or not Lau Yuen Yeung ipso facto became a Filipino
citizen upon her marriage to a Filipino citizen.
HELD:
Under Section 15 of Commonwealth Act 473, an alien woman
marrying a Filipino, native born or naturalized, becomes
ipso facto a Filipina provided she is not disqualified to be a
citizen of the Philippines under Section 4 of the same law.
Likewise, an alien woman married to an alien who is
subsequently naturalized here follows the Philippine
citizenship of her husband the moment he takes his oath as
Filipino citizen, provided that she does not suffer from any of
the disqualifications under said Section 4. Whether the alien
woman requires to undergo the naturalization proceedings,
Section 15 is a parallel provision to Section 16. Thus, if the
widow of an applicant for naturalization as Filipino, who dies
during the proceedings, is not required to go through a
naturalization proceedings, in order to be considered as a
Filipino citizen hereof, it should follow that the wife of a living
Filipino cannot be denied the same privilege.
This is plain common sense and there is absolutely no
evidence that the Legislature intended to treat them
differently. As the laws of our country, both substantive and
procedural, stand today, there is no such procedure (a
substitute for naturalization proceeding to enable the alien
wife of a Philippine citizen to have the matter of her own
citizenship settled and established so that she may not have
to be called upon to prove it everytime she has to perform an
act or enter into a transaction or business or exercise a right
reserved only to Filipinos), but such is no proof that the

citizenship is not vested as of the date of marriage or the


husband's acquisition of citizenship, as the case may be, for
the truth is that the situation obtains even as to native-born
Filipinos. Everytime the citizenship of a person is material or
indispensible in a judicial or administrative case. Whatever the
corresponding court or administrative authority decides
therein as to such citizenship is generally not considered as
res adjudicata, hence it has to be threshed out again and
again as the occasion may demand. Lau Yuen Yeung, was
declared to have become a Filipino citizen from and by virtue
of her marriage toMoy Ya Lim Yao al as Edilberto Aguinaldo
Lim, a Filipino citizen of 25 January 1962.

Coquilla v. Comelec, G.R. No. 151914, July 31,


2002

COQUILLA VS COMELEC
G.R. No. 151914
FACTS:
Coquilla was born on 1938 of Filipino parents in Oras, Eastern
Samar. He grew up and resided there until 1965, when he was
subsequently naturalized as a U.S. citizen after joining the US
Navy. In 1998, he came to the Philippines and took out a
residence certificate, although he continued making several
trips to the United States.
Coquilla eventually applied for repatriation under R.A. No.
8171 which was approved. On November 10, 2000, he took his
oath as a citizen of the Philippines.
On November 21, 2000, he applied for registration as a voter
of Butunga, Oras, Eastern Samar which was approved in 2001.
On February 27, 2001, he filed his certificate of candidacy
stating that he had been a resident of Oras, Eastern Samar for
2 years.
Incumbent mayor Alvarez, who was running for re-election
sought to cancel Coquillas certificate of candidacy on the
ground that his statement as to the two year residency in Oras
was a material misrepresentation as he only resided therein

for 6 months after his oath as a citizen.


Before the COMELEC could render a decision, elections
commenced and Coquilla was proclaimed the winner. On July
19, 2001, COMELEC granted Alvarez petition and ordered the
cancellation of petitioners certificate of candidacy.
ISSUE:
Whether or not Coquilla had been a resident of Oras, Eastern
Samar at least on year before the elections held on May 14,
2001 as what he represented in his COC.

registry of Tai Shun Estate Ltd. filed in Hongkong sometime in


April 1980.
The CID detained Yu pending his deportation case. Yu, in turn,
filed a petition for habeas corpus. An internal resolution of 7
November 1988 referred the case to the Court en banc. The
Court en banc denied the petition. When his Motion for
Reconsideration was denied, petitioner filed a Motion
for Clarification.
ISSUE:

RULING:
No. The statement in petitioners certificate of candidacy that
he had been a resident of Oras, Eastern Samar for two years
at the time he filed such certificate is not true. The question is
whether the COMELEC was justified in ordering the
cancellation of his certificate of candidacy for this reason.
Petitioner made a false representation of a material fact in his
certificate of candidacy, thus rendering such certificate liable
to cancellation. In the case at bar, what is involved is a false
statement concerning a candidates qualification for an office
for which he filed the certificate of candidacy. This is a
misrepresentation of a material fact justifying the cancellation
of petitioners certificate of candidacy. The cancellation of
petitioners certificate of candidacy in this case is thus fully
justified.

Yu vs Santiago, 169 SCRA 364 (1989)

YU vs. DEFENSOR-SANTIAGO
GR No. L-83882, January 24, 1989
FACTS:
Petitioner Yu was originally issued a Portuguese passport in
1971. On February 10, 1978, he was naturalized as a
Philippine citizen. Despite his naturalization, he applied for and
was issued Portuguese Passport by the Consular Section of the
Portuguese Embassy in Tokyo on July 21, 1981. Said Consular
Office certifies that his Portuguese passport expired on 20 July
1986. He also declared his nationality as Portuguese in
commercial documents he signed, specifically, the Companies

Whether or not petitioners acts constitute renunciation of his


Philippinecitizenship
HELD:
Express renunciation was held to mean a renunciation that is
made known distinctly and explicitly and not left to inference
or implication. Petitioner, with full knowledge, and legal
capacity, after having renounced Portuguese citizenshipupon
naturalization as a Philippine citizen resumed or reacquired his
prior status as a Portuguese citizen, applied for a renewal of
his Portuguese passport and represented himself as such in
official documents even after he had become a naturalized
Philippine citizen. Such resumption or reacquisition of
Portuguesecitizenship is grossly inconsistent with his
maintenance of Philippine citizenship.
While normally the question of whether or not a person has
renounced his Philippine citizenship should be heard before a
trial court of law in adversaryproceedings, this has become
unnecessary as this Court, no less, upon the insistence of
petitioner, had to look into the facts and satisfy itself on
whether or not petitioner's claim
to continued Philippine citizenship is meritorious.
Philippine citizenship, it must be stressed, is not a commodity
or were to be displayed when required and suppressed when
convenient.

Frivaldo vs. Comelec, 174 SCRA 245 (1989)

FRIVALDO VS COMELEC
G.R. No. 87193, 23 June 1989 [Naturalization; Reacquisition]
FACTS:
Juan G. Frivaldo was proclaimed governor of the province of
Sorsogon and assumed office in due time. The League of
Municipalities filed with the COMELEC a petition for the
annulment of Frivaldo on the ground that he was not a Filipino
citizen, having been naturalized in the United States.
Frivaldo admitted the allegations but pleaded the special and
affirmative defenses that he was naturalized as American
citizen only to protect himself against President Marcos during
the Martial Law era.
ISSUE:
Whether or not Frivaldo is a Filipino citizen.
RULING:
No. Section 117 of the Omnibus Election Code provides that a
qualified voter must be, among other qualifications, a citizen
of the Philippines, this being an indispensable requirement for
suffrage under Article V, Section 1, of the Constitution.
He claims that he has reacquired Philippine citizenship by
virtue of valid repatriation. He claims that by actively
participating in the local elections, he automatically forfeited
American citizenship under the laws of the United States of
America. The Court stated that that the alleged forfeiture was
between him and the US. If he really wanted to drop his
American citizenship, he could do so in accordance with CA
No. 63 as amended by CA No. 473 and PD 725. Philippine
citizenship may be reacquired by direct act of Congress, by
naturalization, or by repatriation.

Frivaldo vs. Comelec, 257 SCRA 727 (1996)

FRIVALDO VS. COMELEC (1996)


G.R. No. 120295, June 28 1996, 257 SCRA 727

FACTS:
Juan G. Frivaldo ran for Governor of Sorsogon again and won.
Raul R. Lee questioned his citizenship. He then petitioned
for repatriation under Presidential Decree No. 725 and was
able to take his oath of allegiance as a Philippine citizen.
However, on the day that he got his citizenship, the Court had
already ruled based on his previous attempts to run as
governor and acquire citizenship, and had proclaimed Lee,
who got the second highest number of votes, as the newly
elect Governor of Sorsogon.
ISSUE:
Whether or not Frivaldos repatriation was valid.
HELD:
The Court ruled his repatriation was valid and legal and
because of the curative nature of Presidential Decree No. 725,
his repatriation retroacted to the date ofthe filing of
his application to run for governor. The steps to reacquire
Philippine Citizenship by repatriation under Presidential Decree
No. 725 are: (1) filing theapplication; (2) action by the
committee; and (3) taking of the oath of allegiance if
the application is approved. It is only upon taking the oath of
allegiance that theapplicant is deemed ipso jure to have
reacquired Philippine citizenship. If the decree had intended
the oath taking to retroact to the date of the filing of
theapplication, then it should not have explicitly provided
otherwise. He is therefore qualified to be proclaimed governor
of Sorsogon.

Altarejos v. Comelec, G.R. No. 163256. Nov. 10,


2004)

FRIVALDO VS. COMELEC (1996)


G.R. No. 120295, June 28 1996, 257 SCRA 727
FACTS:
Juan G. Frivaldo ran for Governor of Sorsogon again and won.
Raul R. Lee questioned his citizenship. He then petitioned
for repatriation under Presidential Decree No. 725 and was
able to take his oath of allegiance as a Philippine citizen.

However, on the day that he got his citizenship, the Court had
already ruled based on his previous attempts to run as
governor and acquire citizenship, and had proclaimed Lee,
who got the second highest number of votes, as the newly
elect Governor of Sorsogon.
ISSUE:
Whether or not Frivaldos repatriation was valid.
HELD:
The Court ruled his repatriation was valid and legal and
because of the curative nature of Presidential Decree No. 725,
his repatriation retroacted to the date ofthe filing of
his application to run for governor. The steps to reacquire
Philippine Citizenship by repatriation under Presidential Decree
No. 725 are: (1) filing theapplication; (2) action by the
committee; and (3) taking of the oath of allegiance if
the application is approved. It is only upon taking the oath of
allegiance that theapplicant is deemed ipso jure to have
reacquired Philippine citizenship. If the decree had intended
the oath taking to retroact to the date of the filing of
theapplication, then it should not have explicitly provided
otherwise. He is therefore qualified to be proclaimed governor
of Sorsogon.

Tabasa v. Court of Appeals, G.R. No. 125793,


August 29, 2006

TABASA VS CA
G.R. No. 125 793, 29 August 2006 [Naturalization;
Reacquisition; R.A. No. 8171]
FACTS:
When he was 7 years old, Joevanie A. Tabasa acquired
American citizenship when his father became a naturalized
citizen of the US. In 1995, he arrived in the Philippines and was
admitted as "balikbayan"; thereafter, he was arrested and
detained by the agent of BIR. Th Consul General of the US
embassy of Manila filed a request with the BID that his
passport has been revoked and that Tabasa had a standing
warrant for several federal charges against him.
Petitioner alleged that he acquired Filipino citizenship by
repatriation in accordance with the RA No. 8171, and that

because he is now a Filipino citizen, he cannot be deported or


detained by the BID.
ISSUE:
Whether or not he has validly reacquired Philippine citizenship
under RA 8171 and therefore, is not an undocumented alien
subject to deportation.
RULING:
No. Petitioner is not qualified to avail himself of repatriation
under RA 8171. The only person entitled to repatriation under
RA 8171 is either a Filipino woman who lost her Philippine
citizenship by marriage to an alien, or a natural-born Filipino,
including his minor children who lost Philippine citizenship on
account of political or economic necessity.
Petitioner was already 35 years old when he filed for
repatriation. The act cannot be applied in his case because he
is no longer a minor at the time of his repatriation in 1996. The
privilege under RA 8171 only belongs to children who are of
minor age at the time of filing of the petition for repatriation.

Angat v. Republic, GR. No. 132244, September


14,1999

FACTS:
Gerardo Angat, a natural born Filipino citizen, asked to regain
his status as a Philippine citizen before the RTC Marikina. RTC
allowed him to take his Oath of Allegiance on October 3, 1996
and the following day, the RTC declared him as citizen of the
Philippines pursuant to R.A. No. 8171.
OSG filed a Manifestation and Motion in March 1997, asserting
that the petition should have been dismissed by the court for
lack of jurisdiction.
ISSUE:
Whether or not the RTC has jurisdiction in deciding over
repatriation case.
RULING:
No. A petition for repatriation should be filed with the Special
Committee on Naturalization and not with the RTC which has
no jurisdiction.Therefore, the court's order was null and void.

RA No. 8171, which has lapsed into law on October 23 1995, is


an act providing for repatriation of Filipino women who have
lost their Philippine citizenship by marriage to aliens and of
natural-born Filipinos who have lost the Philippine citizenship
on account of political or economic necessity.
Moreover, petitioner was incorrect when he initially invoked RA
965 and RA 2630, since these laws could only apply to persons
who had lost their Philippine citizenship by rendering service
to, or accepting commission in, the armed forces of an allied
country or the armed forces of the US, a factual matter not
alleged in his petition. Parenthetically, under these statutes,
the person desiring to reacquire his Philippine citizenship
would not even required to file a petition in court; all he had to
do is to take an Oath of Allegiance to the Republic of the
Philippines and to register the said oath with the proper civil
registry.

Jao vs. Republic 121 SCRA 358 (1983)

FACTS:
The petitioner filed a petition in the CFI of Davao for
repatriation under Commonwealth Act No. 63, as amended,
alleging therein that although her father was Chinese, she was
a citizen of the Philippines because her mother was a Filipina
who was not legally married to her Chinese husband; that
she lost her Philippine citizenship when she married Go Wan, a
Chinese; and that Go Wan died in September 6, 1962.
Petitioner further alleged that her illiterate mother
erroneously registered her as an alien with the Bureau
ofImmigration, by virtue of which she was issued Alien
Certificate of Registration. The petition contains no prayer for
relief.
The petition was not published, but notice thereof was served
on the Provincial Fiscal who appeared at the hearing but
presented no evidence. After receiving the evidence of the
petitioner, the trial court issued an Order declaring the
petitioner as "judicially repatriated," and ordering the
cancellation of her alien certificate of registration. The
Provincial Fiscal, in behalf of the Republic of the Philippines,
took this appeal.

ISSUE:
WON one may be repatriated by judicial proceedings
HELD:
The proceedings taken in the trial court are
a complete nullity. There is no law requiring or
authorizing that repatriation should be effected by a
judicial proceeding. All that is required for a female
citizen of the Philippines who lost her citizenship to an
alien to reacquire her Philippine citizen, upon the
termination of her marital status, "is for her to take
necessary oath of allegianceto the Republic of the
Philippines and to register the said oath in the proper
civil registry" (Lim vs. Republic, 37 SCRA 783).
Moreover, the petitioner's claim of Philippine citizenship
prior to her marriage for being allegedly an illegitimate
child of a Chinese father and a Filipino mother may not
be established in an action where the mother or her
heirs are not parties (Tan Pong vs. Republic, 30 Phil.
380). It is the consistent rule in this jurisdiction
that Philippine citizenship may not be declared in a nonadversary suit where the persons whose rights are
affected by such a declaration are not parties, such as
an action for declaratory relief (Tiu Navarro vs.
Commissioner of Immigration, 107 Phil. 632) a petition
for judicial repatriation (Lim vs. Republic, supra), or an
action to cancel registration as an alien (Tan vs.
Republic, L-16108, Oct. 31, 1961). As was held in Lim
vs. Republic, supra, "there is no proceeding established
by law or the rules by which any person claiming to be a
citizen may get a declaration in a court of justice to that
effect or in regard to his citizenship."
Note: Although Jaos citizenship prior to her marriage to
a Chinese husband needed judicial confirmation, the
process of repatriation itself (i.e., of reacquiring that
citizenship) involves a purely administrative proceeding.

Nicholas-Lewis v. Comelec, G.R. No. 162759,


August 4, 2006
Facts:
Petitioners were dual citizens by virtue of RA 9225.
Petitioners sought to avail their right of suffrage under
RA 9189 or the Overseas Absentee Voting Act of 2003.
Comelec, however, did not allow petitioners to vote in
the 2004 election, reasoning the petitioners faield to
comply with the requirement of 1-year residency prior
the elections as provided for under Article 5, Sec 1 of
the Constitution.
Issue:
Whether or not petitioners may participate in the
election sans the compliance of the 1 year residency.

Respondent Emilio Lito Osmena filed his certificate of


candidacy before the COMELEC as the Governor of Cebu
Province. Aznar, herein petitioner, as the representative of the
Cebu PDP- Provincial council and as the incumbent Chairman
of such, filed a petition against the respondent before the
Comelec contending that he should be disqualified because
he is not a Filipino citizen, instead an American citizen.
Petitioner filed a Formal Manifestation showing a Certificate
issued by the Immigration and Deportation Commissioner
Miriam Defensor Santiago that the respondent as an American
Citizen is a holder of Alien Certificate of Registration and
Immigrant Certificate of Residence. The Comelec en banc
ordered the Board of Canvassers to continue canvassing but to
suspend the proclamation upon the filing the motion of herein
respondent for the Temporary Restraining Order to the Cebu
Provincial Board of Canvassers from tabulation and
proclamation of the respondent until the resolution of said
petition.

Ruling:
The Court held that those who retained or reacquired
their citizenship under RA 9225 may exercise their right
to vote under the Overseas Absentee Voting Act of
2003, RA 9189.

Private respondent alleged that he is a Filipino Citizen that he


is the legitimate son of Dr. Emilio D. Osmena, the latter being
the son of the late President Sergio Osmena.He also claimed
that he has been continuously residing in the Philippines since
birth and he has not gone out of the country for more than six
months. Furthermore, he contended that he is a registered
voter of the Philippines since 1965.

Article 5, Section 2 of the Constitution provides for the


exception to the residency requirement in Section 1 of
the same article. The voting mechanism in RA 9189 was
practically set forth to provide a system wherein
Filipinos of dual citizenship and are, at the same time,
not residing in the Philippines are empowered to vote.

COMELEC (FirstDivision) directed the Board of Canvassers to


proclaim the winning candidates. Having obtained the highest
number of votes, private respondent was proclaimed the
Provincial Governor of Cebu.

The Court held that present day duals may now exercise
their right of suffrage provided they meet the
requirements under Section 1, Article V of the
Constitution in relation to R.A. 9189

Whether or not private respondent Emilio Lito Osmena has


lost his Filipino Citizenship and thus be disqualified as a
candidate for the Provincial Governor of Cebu Province.

Aznar vs Comelec, G.R. No. 83820, May 25, 1990


FACTS:

ISSUE:

HELD:
NO. The respondent did not lose his Filipino Citizenship and
thereby qualified as a candidate for the Provincial Governor of
Cebu Province. The petitioner failed to present direct proof

that private respondent had lost his Filipino Citizenship by any


of the modes provided under C.A. No. 63 namely: (1) By
naturalization in a foreign country; (2) By express renunciation
of Citizenship; and (3) By subscribing to an oath of allegiance
to support the Constitution or laws of a foreign country. Thus,
it is clear that private respondent Osmea did not lose his
Philippine citizenship by any of the three mentioned herein
above or by any other mode of losing Philippine Citizenship.

Mercado v. Manzano, 307 SCRA 630 (1999)

FACTS:
Petitioner Ernesto Mercado and Eduardo Manzano were both
candidates for Vice-Mayor of Makati in the May 11, 1998
elections.
Based on the results of the election, Manzano garnered the
highest number of votes. However, his proclamation was
suspended due to the pending petition for disqualification filed
by Ernesto Mercado on the ground that he was not a citizen of
the Philippines but of the United States.
From the facts presented, it appears that Manzano is both a
Filipino and a US citizen.
The Commission on Elections declared Manzano disqualified as
candidate for said elective position.
However, in a subsequent resolution of the COMELEC en banc,
the disqualification of the respondent was reversed.
Respondent was held to have renounced his US citizenship
when he attained the age of majority and registered himself as
a voter in the elections of 1992, 1995 and 1998.
Manzano was eventually proclaimed as the Vice-Mayor of
Makati City on August 31, 1998.
Thus the present petition.
ISSUE:

Whether or not a dual citizen is disqualified to hold public


elective office in the philippines.
RULING:
The court ruled that the phrase "dual citizenship" in R.A. 7160
Sec. 40 (d) and R.A. 7854 Sec. 20 must be understood as
referring to dual allegiance. Dual citizenship is different from
dual allegiance. The former arises when, as a result of the
application of the different laws of two or more states, a
person is simultaneously considered a national by the said
states. Dual allegiance on the other hand, refers to a situation
in which a person simultaneously owes, by some positive act,
loyalty to two or more states. While dual citizenship is
involuntary, dual allegiance is a result of an individual's
volition. Article IV Sec. 5 of the Constitution provides "Dual
allegiance of citizens is inimical to the national interest and
shall be dealt with by law."
Consequently, persons with mere dual citizenship do not fall
under this disqualification. Unlike those with dual allegiance,
who must, therefore, be subject to strict process with respect
to the termination of their status, for candidates with dual
citizenship, it should suffice if, upon the filing of their
certificates of candidacy, they elect Philippine citizenship to
terminate their status as persons with dual citizenship
considering that their condition is the unavoidable
consequence of conflicting laws of different states.
By electing Philippine citizenship, such candidates at the same
time forswear allegiance to the other country of which they
are also citizens and thereby terminate their status as dual
citizens. It may be that, from the point of view of the foreign
state and of its laws, such an individual has not effectively
renounced his foreign citizenship. That is of no moment.
When a person applying for citizenship by naturalization takes
an oath that he renounces his loyalty to any other country or
government and solemnly declares that he owes his allegiance
to the Republic of the Philippines, the condition imposed by
law is satisfied and complied with. The determination whether

such renunciation is valid or fully complies with the provisions


of our Naturalization Law lies within the province and is an
exclusive prerogative of our courts. The latter should apply
the law duly enacted by the legislative department of the
Republic. No foreign law may or should interfere with its
operation and application.

Valles v. Comelec, G.R. 137000, August 9, 2000

Principle of jus sanguinis


How Philippine citizenship is acquired
Effect of filing certificate of candidacy: express
renunciation of other citizenship

The court ruled that the filing of certificate of candidacy of


respondent sufficed to renounce his American citizenship,
effectively removing any disqualification he might have as a
dual citizen. By declaring in his certificate of candidacy that he
is a Filipino citizen; that he is not a permanent resident or
immigrant of another country; that he will defend and support
the Constitution of the Philippines and bear true faith and
allegiance thereto and that he does so without mental
reservation, private respondent has, as far as the laws of this
country are concerned, effectively repudiated his American
citizenship and anything which he may have said before as a
dual citizen.

FACTS:

On the other hand, private respondents oath of allegiance to


the Philippines, when considered with the fact that he has
spent his youth and adulthood, received his education,
practiced his profession as an artist, and taken part in past
elections in this country, leaves no doubt of his election of
Philippine citizenship.

Whether or not Rosalind is an Australian or a Filipino

His declarations will be taken upon the faith that he will fulfill
his undertaking made under oath. Should he betray that trust,
there are enough sanctions for declaring the loss of his
Philippine citizenship through expatriation in appropriate
proceedings. In Yu v. Defensor-Santiago, the court sustained
the denial of entry into the country of petitioner on the ground
that, after taking his oath as a naturalized citizen, he applied
for the renewal of his Portuguese passport and declared in
commercial documents executed abroad that he was a
Portuguese national. A similar sanction can be taken against
any one who, in electing Philippine citizenship, renounces his
foreign nationality, but subsequently does some act
constituting renunciation of his Philippine citizenship.
The petition for certiorari is DISMISSED for lack of merit.

Rosalind Ybasco Lopez was born on May 16, 1934 in Australia


to a Filipino father and an Australian mother. In 1949, at the
age of fifteen, she left Australia and came to settle in the
Philippines, where she later married a Filipino and has since
then participated in the electoral process not only as a voter
but as a candidate, as well. In the May 1998 elections, she ran
for governor but Valles filed a petition for her disqualification
as candidate on the ground that she is an Australian.
ISSUE:

HELD:
The Philippine law on citizenship adheres to the principle of jus
sanguinis. Thereunder, a child follows the nationality or
citizenship of the parents regardless of the place of his/her
birth, as opposed to the doctrine of jus soli which determines
nationality or citizenship on the basis of place of birth.
Rosalind Ybasco Lopez was born a year before the 1935
Constitution took into effect and at that time, what served as
the Constitution of the Philippines were the principal organic
acts by which the United States governed the country. These
were the Philippine Bill of July 1, 1902 and the Philippine
Autonomy Act of Aug. 29, 1916, also known as the Jones Law.
Under both organic acts, all inhabitants of the Philippines who
were Spanish subjects on April 11, 1899 and resided therein
including their children are deemed to be Philippine citizens.
Private respondents father, Telesforo Ybasco, was born on Jan.
5, 1879 in Daet, Camarines Norte.... Thus, under the Philippine
Bill of 1902 and the Jones Law, Telesforo Ybasco was deemed

to be a Philippine citizen. By virtue of the same laws, which


were the laws in force at the time of her birth, Telesforos
daughter, herein private respondent Rosalind Ybasco Lopez, is
likewise a citizen of the Philippines.
The signing into law of the 1935 Philippine Constitution has
established the principle of jus sanguinis as basis for the
acquisition of Philippine citizenship, xxx
So also, the principle of jus sanguinis, which confers
citizenship by virtue of blood relationship, was subsequently
retained under the 1973 and 1987 Constitutions. Thus, the
herein private respondent, Rosalind Ybasco Lopez, is a Filipino
citizen, having been born to a Filipino father. The fact of her
being born in Australia is not tantamount to her losing her
Philippine citizenship. If Australia follows the principle of jus
soli, then at most, private respondent can also claim
Australian citizenship resulting to her possession of dual
citizenship.

You might also like